Сохранен 532
https://2ch.hk/sci/res/237334.html
24 декабря Архивач восстановлен после серьёзной аварии. К сожалению, значительная часть сохранённых изображений и видео была потеряна. Подробности случившегося. Мы призываем всех неравнодушных помочь нам с восстановлением утраченного контента!

Математика, тред 9

 Аноним 07/02/15 Суб 06:59:19 #1 №237334 
14232815598880.jpg
Никогда она мне не давалась, дрянь. Ну разве мне могут пригодится эти чертовы логарифмы и дискриминанты? Разве я буду стоять в магазине и наспех рисовать график, чтобы купить кило сахара? Ненавижу её.

Если конкретно, то вся матика после 6-го класса это чистой воды флуд. Она абсолютно бесполезна, и предназначена чтобы паразитировать на студентах и школьниках. Я считаю что высшая математика, тригонометрия, анализ - жестокая шутка учёных-сухарей.

терпеть не могу матему, я в школе дрожала на этих уроках. И офигела, когда еще и в универе, чуть ли ни на филологическом фак-те у мня была вышка! На черта? Я и там не до чего не дорубалась, сидела тупнем, потом вовсе ходить перестала, зачет передрала у кого-то, как в школе, но я сама искренне пыталась что-то решить, зря время теряла, все равно с ответами не совпадало. А в школе я матему класса до 8 понимала, и то так, поверхностно, ну тогда эт из-за лени было, а в 9 я совсем в неё не вникла, и так до 11 не дошла до всяких формул. Я всё списывала, сдирала...я могла дискриминант посчитать!!! И еще кое-какие формулы заучила, и с трудом могла графики строить, самы фонарные. В ту же кучу у меня полетела геометрия, в ней я вообще потерялась. Мы с одноклассницей сидели вдвоем и тупили. У нас почти никогда домашки не было, я её НИКОГДА не делала, одно дело лень было, а другое, что мне и не в падлу было, да вот только всё равно мозгов не хватало ее делать.

А этим, между прочим, математички страшно бесят...им-то легко призывать кого-то включить мозги. Проблема в том, что я со своими вообще не лажу, и в особенности по части математики. Другое дело, если бы они подробно рассказали, как это делается, ато просто трепаться и требовать каждый горазд.

вот что что, а физ-мат мне никогда не удавался! это не моё, я не получаю кайфа от решения уравнений и задач. иногда смотрю на доску и не понимаю что делать с этим примером! зато в гуманитарии мне цены нет, запоминаю всё на лету и очень легко понимаю весь материал. но алгебра, геометрия., физика и химия, это кранты! это надо понимать, а если не понимаешь, сидеть и брать жопой (это способ : когда понимаешь - берешь головой, когда не понимаешь, но надо - берешь жопой, т.е. усердием, а у меня его нет, и я не заморачиваюсь по этому поводу), и вообще я не думаю, что всё, что мы учим на уроках математики пригодиться нам в будущей жизни, во всяком случае для меня, я никогда не пойду на профессию, которая связана с расчётами, и т.д., а вот творческая работа мне подойдёт!

Бесполезная наука, морально устарела. Чтобы решать примеры нужно держать в голове ВСЁ, что проходил раньше. Сегодня с этой задачей успешно справляются компьютеры. Например, в Excel можно забить формулы, а дальше только ставить цифры. Если не знаешь формул - ищешь в интернете. Человеку остается интерпретировать ответ. Для 21 века более актуально составление алгоритмов.

Сегодняшний прогресс существует благодаря надстройкам на прошлых достижениях. Ничего с нуля сегодня не делается. Надо быть рациональными: машины считают лучше людей, пусть этим и занимаются, совершенствуют технологии, пишут программы и алгоритмы, которые нужны людям, производят технику и самих себя. Развивать мозги можно читая книги/интернет и перенимать опыт других, который поднесен на блюдечке. Математика - такой же архаизм, как ориентация по звездам, труд кузнеца. И такие изобретения, как колесо, обработка железной руды, добыча огня - это было важным катализатором прогресса, но математики там не понадобилось. А чтобы вас не обсчитали в ларьке - носите калькулятор. Лучше ходить в нормальные магазины и не трястись из-за ста рублей, там техника все быстро считает сама. А если готовы рисковать миллионами, бравируя своей способностью оперировать числам и не проверяя себя - вы идиот.

Я ненавижу математику! У меня аналитический склад ума, она мне очень тяжело даётся! В эпоху информационных технологий не вижу необходимость знать математику обычному человеку. Время умеешь посчитать, сдачу в магазине, зарплату, и на этом можно остановиться. В остальных случаях можно воспользоваться калькулятором, который есть сейчас в любом сотовом телефоне и на любом компьютере. И необязательно учить и знать эти проклятые формулы!

Эта наука отупляет на весь свой нулевой корень в бесконечной степени... Ну чего интересного ломать свой мозг от объективно не существующие абстрактные идеи?! Мне приходится осиливать этот плод логической деградации только из-за любви к физике и другим естественным наукам. Чего уж поделать, если этот идиотизм является нучным языком... Но фапать на математику ради математики... - могут только те, которым просто занятся больше нечем.

я ненавижу математику!!!я ее не понимаю, она для меня скучна и неинтересна свой последовательностью и логикой,я за творчество, математика не для меня.

Предыдущий утоп там: >>230214
Аноним 07/02/15 Суб 07:50:13 #2 №237338 
>>237334
https://arhivach.org/thread/56479/
Аноним 07/02/15 Суб 08:51:49 #3 №237339 
Без знаний прошлого невозможно изобести чего-нибудь нового. Вот и будем дальше сидеть в параше без математики/физики/биологии/химии.
Математика нужна не только для подсчётов и решения уравнений, но и для развития логики, ведь, в математике всё логично и одно вытекает из другого. Да и для творчества тут места очень даже.
Аноним 07/02/15 Суб 09:42:35 #4 №237343 
>>237339
>ведь, в математике всё логично и одно вытекает из другого
хуита
Аноним 07/02/15 Суб 14:04:07 #5 №237360 
>>237334
Ты просто тупое быдло, тебе лучше на повара учиться.
Аноним 07/02/15 Суб 15:43:25 #6 №237385 
>>237334
толсто
Аноним 07/02/15 Суб 17:37:09 #7 №237390 
>>237334
Тупорылая гуманитаристическая пизда (если не просто троль и копипастишь) , я с такими как ты достаточно общался на сайтах - ты безнадежна, и не понимаешь этого по своей природной быдлости. Ты думаешь, что моск нужен чтобы запомнить кучу говна сказанного по тому же самому поводу - и тебе даже не приходит в голову сравнить аргументы трех уродов чтобы понять, что все трое несут полную хуйню.
Математику надо понимать, а не помнить - лишь пиздомозглый гуманитарий думает обратное .
Аноним 07/02/15 Суб 17:40:20 #8 №237391 
>>237334
вечер в хату, подскажите как доказывается неравенство треугольника для метрики Хаусдорфа
Аноним 07/02/15 Суб 17:50:23 #9 №237392 
>>237360
>>237385
>>237390
Откуда вы вылезли, имбецилы? Математики хуевы, блядь. Детишки тупорылые.
Аноним 07/02/15 Суб 17:56:09 #10 №237393 
>>237392
Иди нахуй, троль тупая. Скучно...
Аноним 07/02/15 Суб 18:26:12 #11 №237396 
>>237334
Удваиваю. Для школьников достаточно базовых знаний арифметики и геометрии. Всё что сверх этого - пускай упоротые наукобляди дрочат на своих кафедрах.
Аноним 07/02/15 Суб 18:33:31 #12 №237400 
>>237396
Ты понимаешь, что эти базовые знания можно дать лет до 10? А что потом с пиздюками делать?
Аноним 07/02/15 Суб 18:42:57 #13 №237401 
>>237400
пиздюков пиздить сраными тряпками, очевидно же
а нормальных детей отдавать во всякие там пиздесят седьмые
Аноним 07/02/15 Суб 20:32:18 #14 №237433 
>>237400
Пускай в доту рубятся. Один хрен, всё забудут после института. Так пускай лучше проведут время с удовольствием.
Аноним 07/02/15 Суб 20:36:58 #15 №237436 
14233306184550.jpg
На самом деле все правильно. Тупая пизда должна быть тупой пиздой на максимум. Соответственно математик и способный ученик должен учиться отдельно от тупой пизды, чтобы не тормозить вместе с тупарями и одновременно чтобы тупари не страдали от чрезмерной напруги на мозг.
Вообще идеально бы иметь школы по типу полного разделения по способностям.
Умные учатся с умными.
Тупари и быдло с такими же соответственно.
Аноним 07/02/15 Суб 21:49:54 #16 №237450 
>>237334
в своё время при работе офисным планктоном сумел неплохо рационализировать свой труд, подключив к обычным сложениям/вычитаниям знания о степенях, логарифмах, арифметических и геометрических последовательностях, квадратные уравнения. Уже чуть после заинтересовался программированием и там дополнительно понадобились знания тригонометрии, синусы, косинусы, матрицы. Программирование мне также помогло в работе, некоторые вещи сумел внедрить, получил повышение.
Вот производные, интегралы и пределы я плохо понимаю как можно применить.
Аноним 07/02/15 Суб 21:58:21 #17 №237452 
>>237450
>Интегралы
>не знает как применить
Да ты охуел!
Аноним 07/02/15 Суб 22:07:02 #18 №237454 
>>237452
я знаю что такое интегралы, я не знаю зачем мне в жизни может потребоваться вычислять площадь под криволинейной фигурой... Ну если только как в байке про пистолет и туалет...
Аноним 08/02/15 Вск 12:29:54 #19 №237509 
Что анон думает про сочинения В. Босса? http://www.google.ru/url?q=http://www.alleng.ru/d/math/math153.htm&sa=U&ei=SSzXVIKCN6axygPKzILABw&ved=0CAsQFjAA&usg=AFQjCNHT8r3W1o_enxrYmiOZqoix-lgEKQ
Аноним 08/02/15 Вск 12:39:15 #20 №237512 
>>237509
Арон думает, что это комиксы для тех, кто уже в теме. В крайнем случае - полезно, чтобы получить общее представление о каком-то разделе математики.
Аноним 08/02/15 Вск 13:23:17 #21 №237518 
>>237509
> Для нормального изучения любого математического предмета необходимы по крайней мере четыре ингредиента:
> 1) живой учитель;
Да ну нахуй, я пишу %subject-name% undergraduate book, скачиваю и решаю задачи. Конечно, обсуждать с кем-то было бы интереснее, но мне и так норм.
Аноним 08/02/15 Вск 14:18:06 #22 №237534 
>>237518
Так учитель нужен, чтобы если застрял, быстрее понять тему. Это как минимум продуктивнее.
Аноним 08/02/15 Вск 16:07:34 #23 №237554 
>>237334
>то вся матика после 6-го класса это чистой воды флуд
Совершенно верно, это сорт вероблядства. Закон божий отменили, но СВЯТО МЕСТО ПУСТО НЕ БЫВАЕТ. Совершенно оторвано от реальности, используются специальные ритуальные слова и фразы, надо тупо запоминать и повторять как молитвы.
>>237339
>в математике всё логично и одно вытекает из другого
Для утенка которому вдолбили.
Аноним 08/02/15 Вск 17:03:44 #24 №237567 
14234042248070.png
>>237509
Аноним 08/02/15 Вск 17:43:18 #25 №237574 
Самый быстрый путь возобновить теоретическую базу по математике для инженера или физика (не теоретика).
1. Сканави - Элементарная математика
2. Курант - Что такое математика? (содержит начала анализа, которые пропущены в учебнике Сканави, плюс немного отвлеченных разделов по топологии, теории множеств и т.д.).
3. Мышкис - Лекции по высшей математике.
4 (опционально). Сигорский - Математический аппарат инженера.

Каковы ваши предложения/уточнения?
Аноним 08/02/15 Вск 17:47:44 #26 №237575 
>>237574
Если речь о математике именно для физика или инженера и именно о том, чтобы вспомнить, то нафига все это? Какие- нибудь справочники по высшей и элементарной математике типа Выгодского и все.
Аноним 08/02/15 Вск 17:58:16 #27 №237576 
>>237575
Ну хз, я вот в институте учился на отъебись - то есть, только чтобы вытянуть на стипендию, поехать на книжный рынок, накупить фэнтезятины и еще месяц поплавать на волнах эскапизма. И хотя я все 6 лет получал стипендию, а по вышке у меня было отлично и хорошо, но теорию я не помню (потому что все задачи решались "по примерам", а не по глубокому пониманию теории). А вот сейчас стало интересно.
Аноним 08/02/15 Вск 18:23:29 #28 №237578 
14234090091270.png
Недавно задумался (пикрилейтед) - а что если действительно так?
Жизнь, когда увлекаешься математикой и проводишь почти все время с ней, действительно становится скучна, но я всегда считал, что причина этого лежит в самом окружающем мире.

Учитывая контингент вышеотписавшихся, не особо надеюсь найти отклик, однако похуй, может ответите что-то интересное, аноны.
Аноним 08/02/15 Вск 18:34:09 #29 №237579 
>>237576
Ну смотри сам, лол. Если погружаться в теоретическую физику задачи не стоит, то особо много тебе не нужно- анализ, каая- нибудь векторная алгебра, вот это все.
Вобщем- то, если с "вышкой" у тебя было все хорошо и тебе нужно вспомнить физику на уровне инженера, то я вообще не не вижу особого смысла задрачивать как- то отдельно математику. Просто бери того же Сивухина и если будут какие- то непонятки с математикой лезь в справочник.
Аноним 08/02/15 Вск 18:58:15 #30 №237583 
>>237534
Ну хз, у меня для таких вещей solution manual.
Аноним 08/02/15 Вск 19:00:15 #31 №237584 
>>237578
Давай ты скажешь имя того, кто пишет там от первого лица, мы загуглим, чего он добился, и все станет на свои места.

А вообще да, чистые математики очень оторваны от мира, в физике не бум-бум, например.
Аноним 08/02/15 Вск 19:04:43 #32 №237585 
>>237584
Это же воспоминания Гротендика.
Аноним 08/02/15 Вск 19:05:55 #33 №237586 
>>237584
>урожаи и посевы
>имя автора
>загуглим
>math thread
Ну офигеть.
Аноним 08/02/15 Вск 19:10:33 #34 №237587 
>>237578
Возможно, если математика не превращается в вещь в себе и для себя, она меньше сушит мозги. Если человек занимается каими-то прикладными вещами, скажем. Топологи или алгебраисты — в среднем больше шиза, чем те, кто по дифурам или теории вероятности. Во всяком случае на первый взгляд.
Аноним 08/02/15 Вск 19:14:18 #35 №237588 
>>237586
>урожаи и посевы
Лел, я сейчас "Сады и дороги" Юнгера читаю.
Аноним 08/02/15 Вск 19:40:36 #36 №237590 
>>237584
пиздос, у меня во дворе, за не знания такого просто убивают нахуй
Аноним 08/02/15 Вск 19:47:55 #37 №237591 
14234140759360.jpg
Эй, первокуры, а кто из вас слышал про Гротендика до того, как он умер?
Аноним 08/02/15 Вск 19:53:46 #38 №237592 
>>237591
А в чем разница? Мейнстримность Гротендика влияет на элитарность обсуждающего его личность?

Гротендик - достаточно нетипичный и живой образ. Естественно, что им будут вдохновляться младшекуры. Да и плохо ли.
Аноним 08/02/15 Вск 19:56:56 #39 №237593 
>>237591
Я слышал о нём до того, как он родился.
Аноним 08/02/15 Вск 19:57:37 #40 №237594 
>>237592
Мне смешно видеть эту гротендиковскую истерию после того, как известие о его смерти растиражировали СМИ. Теперь каждый воннабиматематик считает своим долгом ввернуть его в обсуждение. Хотя полгода назад никто из здесь присутствующих (включая меня, разумеется) и не подозревал о его существовании и ОГРОМНОМ вкладе в мировую математику.
Аноним 08/02/15 Вск 20:00:35 #41 №237595 
>>237594
Говори за себя. Мне ГРОТЕНДИК был известен ещё в школе.
Аноним 08/02/15 Вск 20:03:18 #42 №237596 
>>237594
Ох лол.
Что является признаком ОГРОМНОСТИ вклада в мировую математику?
Аноним 08/02/15 Вск 20:04:56 #43 №237597 
>>237595
Угу-угу. Мне тоже. В детском саду только. >>237593 вон вообще его рождение предвидел.

И смех, и грех. Крестьяне не могут в научное мировоззрение и по заложенной в генах привычке молятся на идолов. Раньше у вас был Перельман, теперь вот нового возвели, спасибо Ленте.ру. А не помер бы Гротендик - продолжали бы мусолить поистрепавшегося Перельмана.
Аноним 08/02/15 Вск 20:05:09 #44 №237598 
>>237594
Да нет, мысли о том, что Гротендик достаточно интересная фигура существовали задолго до его смерти. В более узких кругах, но это особого значения, думаю, не имеет.
Аноним 08/02/15 Вск 20:05:38 #45 №237599 
>>237596
На Ленте.ру когда напишут - значит, ОГРОМНЫЙ.
Аноним 08/02/15 Вск 20:07:12 #46 №237600 
>>237598
Я этого и не отрицаю, разумеется его знали. Но не нынешние первокуры, которые сейчас с пеной у рта рассказывают, как зачитывались "Урожаями" в детском саду.
Аноним 08/02/15 Вск 20:10:14 #47 №237601 
>>237597
Вспомнилось из интервью Манина
"пока есть такие люди, как Перельман и Гротендик, мы будем помнить наш идеал."

А что плохого? Стремление иметь больше общего с некоторым образом, как правило, части своего же идеализированного суперэго. Понятно, что это сладкая почва для тех, кто нихуя не будет делать на самом деле, но это другой вопрос уже.
Аноним 08/02/15 Вск 20:11:37 #48 №237602 
>>237599
Ну, то что ты узнал об этом с Ленты, все же говорит больше о тебе, а не о Гротендике.
Аноним 08/02/15 Вск 20:13:57 #49 №237603 
>>237602
Я и говорю не о Гротендике, а о себе. И о тебе, ведь ты тоже с ленты узнал. Как и все остальные ИТТ.
Аноним 08/02/15 Вск 20:15:03 #50 №237604 
>>237601
>А что плохого?
Да расслабься, это просто очередной у мамы нитакой как фсе. Если о ком- то много говорят, восхищаются, значит нужно обязательно его, хоть чуть чуть, но подобосрать. Такая себе форма самоутверждения.
Аноним 08/02/15 Вск 20:16:32 #51 №237605 
>>237603
>эти проекции
Ну ясно.
Аноним 08/02/15 Вск 20:17:16 #52 №237606 
>>237603
>а о себе
Вот именно.
Аноним 08/02/15 Вск 20:19:12 #53 №237607 
>>237601
Ничего плохого, если человек говорит честно: "Надо же, а я и не слышал ничего о нем раньше. Решил восполнить пробел, прочитаю Урожаи и Посевы на досуге."

А вот это вот "Ты что, не знаешь Гротендика, самого выдающегося математика 20-го века? Да он же первым <пара терминов, прочитенных на странице Гротендика в вики>. Я еще в школе им зачитывался!" - это говно в голове.
Аноним 08/02/15 Вск 20:21:22 #54 №237609 
>>237607
>"Ты что, не знаешь Гротендика, самого выдающегося математика 20-го века?
И где же ты углядел такое в этом треде?
Аноним 08/02/15 Вск 20:21:51 #55 №237610 
>>237605
>>237606
Еще немного, и я поверю, что ты знал о нем раньше.

Хм, а впрочем нет. Не поверю.
Аноним 08/02/15 Вск 22:05:41 #56 №237616 
>>237584
>чего он добился
Сперва добейся! лол
Аноним 08/02/15 Вск 22:06:35 #57 №237617 
>>237603
Ты че охуел, ньюфажина? Его еще в первых тредах поминали, как одного из авторов трактата.
Аноним 08/02/15 Вск 22:48:26 #58 №237623 
>>237578
Охлол, сайентач открывает для себя понятие задротства.
Аноним 08/02/15 Вск 23:38:04 #59 №237634 
>>237623
Дело не в задротстве как таковом.

Сама по себе учеба, такое общее задротничество, не влияет особенно на восприятие мира. Однако я помню, каким был я перед углублением в математику (под этим я подразумеваю уделять ей все свободное время и зачастую несвободное). У меня был абонемент в качалку, симпатичная и неглупая девушка, с которой было много радужных планов на будущее, были друзья и огромное количество тех, с кем иногда можно провести время.

Постепенно все становилось все более и более скучным, я бросил тренировки, потому что трудно думать после тренировок, я перестал общаться с девушкой, потому что это долго и не особенно интересно, контакты сузились до (едва) двух человек, поэтому сейчас, когда появилась необходимость услышать чье-то мнение, я пишу сюда - почти в пустоту.

Я уже 5 дней почти не думаю о математике, а думаю о своей жизни. Когда-нибудь я утрачу возможность заниматься математикой так же, как сейчас (человек с возрастом тупеет), и если все будет идти так же как сейчас, у меня не останется ничего. И легко можно было вернуть все назад, наверно, но я знаю, что будет невыносимо скучно.
Аноним 09/02/15 Пнд 00:11:13 #60 №237638 
>>237634
Достопочтенный господин, сколько вам лет?
Аноним 09/02/15 Пнд 00:21:18 #61 №237644 
>>237518>>237583
Удачи с изучением чего-то серьезнее программы первого курса. Нет, конечно, можно, но сложность нетривиально возрастает.
>>237594
Скажи честно, ты никогда не был в тусовке приличных математиков? Блядь, мне про него лекцию читали даже в олимпиадной школе. А уж в НМУ вообще сложно неделю проходить и не узнать его биографии.
Аноним 09/02/15 Пнд 00:21:33 #62 №237645 
>>237638
Мне немногим больше 20.
Аноним 09/02/15 Пнд 01:25:45 #63 №237671 
14234343458310.jpg
>>237578
>причина этого лежит в самом окружающем мире.
Любая тема, на которой ты становишься помешанным, бьет прежде всего по соц связям.
Начиная от наркоты и заканчивая математикой.
Но если ты не будешь раскачивать тему на максимум, у тебя вряд-ли получится стать лучшим/получить достойный результат.
Охуенно, когда твой социальный круг в курсе чем ты занят и поддерживает тебя. Но таких людей еще поискать надо.
Меня допустим мои "нормальные" друзья тоже почти все нахуй послали.
Остались только всякие задроты да двощ. Но я ценю, что хотя-бы они у меня есть.
Аноним 09/02/15 Пнд 01:36:14 #64 №237674 
>>237671
>Но я ценю, что хотя-бы они у меня есть.
Пошел нахуй отсюда. Лучше бы бабу нашел, сразу бы дурь из башки выбилась.
Аноним 09/02/15 Пнд 01:48:12 #65 №237675 
>>237674
>сразу бы дурь из башки выбилась.
Ага, вместе с мозгом.
Аноним 09/02/15 Пнд 02:41:51 #66 №237684 
Почему математики так дрочат на детей? Все математики педофилы? Гротендик дрочит в урожаях на детей, Колмогоров дрочил на молодые таланты и сам признавался, что его психологический возраст - 14 лет, Арнольд себя детишками окружал. Почему чтобы стать математиком надо превратиться в ебаного морального урода?
Аноним 09/02/15 Пнд 02:42:50 #67 №237685 
>>237684
Задротство = инфантильность.
sageАноним 09/02/15 Пнд 02:51:45 #68 №237688 
>>237684
С педофилией в /po/рашу прошествуй, сезонные моральные уродства там обсудишь.
Аноним 09/02/15 Пнд 03:26:31 #69 №237696 
Может всё-таки о математике поговорим? Например, мне интересно как классифицировать векторные расслоения над сферами. Кто-нибудь знает?
Аноним 09/02/15 Пнд 03:32:22 #70 №237698 
>>237696
А мне интересны интегральчики под картофанчик гомотопические группы сфер. Кто-нибудь знает?
Начинается,блять, вербитопонтоёбство
Аноним 09/02/15 Пнд 03:38:39 #71 №237699 
>>237698
Гротендик, кстати, в своих "урожаях и посевах" писал об уродских проявлениях снобизма, элитизма и меритократии в этих ваших математических тусовках.
Аноним 09/02/15 Пнд 05:02:51 #72 №237702 
>>237698
Спроси Романа Михайлова, он точно знает, он как раз гомотопическими группами сфер занимается
Аноним 09/02/15 Пнд 07:13:44 #73 №237708 
>>237334
ну и тупой еблет на пике
Аноним 09/02/15 Пнд 18:10:57 #74 №237821 
>>237702
> Роман Михайлов
Аноним 09/02/15 Пнд 21:39:39 #75 №237867 
>>237821
Смотрел, думаю тоже как-нибудь упороться и познать математическую истину.
Аноним 09/02/15 Пнд 23:03:46 #76 №237897 
Я изучаю программу 1-2 курса. Перед тем как читать приведенные в учебнике доказательства, я всегда читаю только формулировку и пытаюсь доказать сам, если не смог, тщательно вникаю и потом воспроизвожу доказательство на бумаге. Так вот, иногда от этого все охуенно упорядочивается в голове и материал начинает казаться очевидным. Но доказательства некоторых теорем состоят из каких-то ебанутых магических трюков (сгруппировать хуй, добавить пизду здесь и вычесть там). Я понимаю, почему то или иное доказательство верно, но я не понимаю, как вообще можно было до такого догадаться. Так вот вопрос: стоит ли мне тщательно обдумывать и воспроизводить доказательства второго типа, или это мусор, никак не способствующий восприятию материала?
Аноним 09/02/15 Пнд 23:08:02 #77 №237899 
14235124825270.png
14235124825291.png
>>237897
Вот пример такого доказательства.
Аноним 09/02/15 Пнд 23:30:37 #78 №237906 
14235138377740.jpg
>>237698
Так что ты говорил про интергальчик под картофанчик?
Аноним 09/02/15 Пнд 23:42:20 #79 №237909 
>>237897
Так иногда бывает. Привыкаешь к такому постепенно и не понимаешь, как такие ранее искусственные приемы уже рождаются в твоем сознании.
С другой стороны, бывают часто совсем технические доказательства, и зачастую можно найти проще, как и, судя по всему, в твоем примере.

Алсо, бывает так, что для того чтобы расколоть крепкое доказательство, нужно не долбить по нему молотком оттуда и сюда, а кинуть его в море рассуждений типа "вокруг да около" и ждать, пока оно станет более мягким для того, чтобы просто взять его в руку и расколоть.
Аноним 09/02/15 Пнд 23:53:08 #80 №237913 
>>237909
Дак че, если разбираться с подобными доказательствами, у меня появится смекалочка? Или не стоит на них время тратить?
Аноним 10/02/15 Втр 00:11:25 #81 №237914 
>>237913
Мне кажется, нет никакого смысла разбираться, если не понимаешь, зачем так было.
Если утверждение кажется важным, лучше подумать о мотивировках.
Аноним 10/02/15 Втр 00:26:45 #82 №237918 
>>237899
Ну до момента с супремумом все понятно и логично. Дальше какая-то еботень. Вообще показывать непрерывность функции (а именно это по большому счету и пытаются сделать) до курса матана, хотя бы азы, хотя бы на пальцах - это какое-то говноедство.
Аноним 10/02/15 Втр 01:16:33 #83 №237924 
Аноны, я тут подумал, что ведь когда-то развитие человеческой математики остановится, так как конечность математики является следствием свойства самого фундамента математики, а именно человеческого сознания. Ведь по сути все это есть постепенная формализация того, что уже глубоко в нас заложено, на нижних уровнях абстракций, на интуитивных уровнях, восприятие пространства, непротиворечивости чего-либо и прочее. В итоге все придет к тому, что компьютеры будут дальше толкать науку, занимаясь отчасти бессмысленными для людей символьными вычислениями, а на выходе мы будем получать вполне себе "живой" результат.
sageАноним 10/02/15 Втр 01:27:28 #84 №237926 
>>237924
Сказал - как в лужу пернул. Пиздуй отсюда, фелософ мамкин.
Аноним 10/02/15 Втр 02:53:03 #85 №237929 
>>237924
Скорее, не формализация интуиции, а ее расширение. Visual system, предназначенная, наверно, для перемещения по местности и воображения, для углубления в свойства пространства (точнее - формы) в занятиях геометрией годится лишь частично.
Аноним 10/02/15 Втр 06:48:49 #86 №237942 
как можно доказать что что для функции [math]e^{-x}=\sum \frac{(-1)(x^{k}}{k!}[/math] что для этой функции, лимит как x идет к бесконечности=0. Но нельзя пользоваться алгеброй( то есть нельзя [math]e^{-x}=\frac{1}{e^x}[/math]?
Аноним 10/02/15 Втр 11:23:56 #87 №238012 
>>237578
Если тебе ничего кроме математики не интересно, то это проблема в тебе а не в математике.
Аноним 10/02/15 Втр 12:13:43 #88 №238050 
>>237334
Ох и бомбануло у меня! Автор ты биомусор,я думаю что математика лежит почти в основе всего
Аноним 10/02/15 Втр 12:51:43 #89 №238068 
14235619039440.jpg
>>238050
Вы меня разочаровываете, математики мамкины
Аноним 10/02/15 Втр 13:40:37 #90 №238088 
>>237926
Поясни, что тебе не нравится.
http://elementy.ru/news?newsid=432401
Математика берет начало именно в низкоуровневом (чувственном, если так можно сказать) восприятии природы.
>>237929
Ну да, вводятся более общие и абстрактные понятия. Вполне возможно, что в какой-то момент мы не сможем в силу "залоченности" мозга продвинуться дальше в объяснении вещей окружающего мира.
Аноним 10/02/15 Втр 13:43:19 #91 №238089 
>>237591>>237607
А кто из вас знает о Тейхмюллере сейчас?
Аноним 10/02/15 Втр 15:56:11 #92 №238106 
>>238089
Я! Теорию Тейхмюллера форсили ещё десятки тредов назад.
Аноним 10/02/15 Втр 16:12:00 #93 №238108 
>>238089
Этим Тейхмюллером был Гротендик?
Аноним 10/02/15 Втр 16:26:57 #94 №238113 
>>238108
Перельман обоим наваляет одной левой.
Аноним 10/02/15 Втр 16:31:34 #95 №238114 
>>238113
А много ли тут первокуров, которые знали о Перельмане до того, как он умер?
Аноним 10/02/15 Втр 17:31:59 #96 №238122 
>>238114
Практически все. А вот знал ли их Перельман?
Аноним 10/02/15 Втр 17:35:21 #97 №238124 
>>238122
Но ведь он не умер.
Аноним 10/02/15 Втр 17:42:13 #98 №238125 
>>238124
>Михаи́л Изра́илевич Перельма́н (20 декабря 1924, Минск — 29 марта 2013, Москва) — советский и российский хирург, известный своими научными работами в области торакальной хирургии. Академик АМН СССР — РАМН.
Аноним 10/02/15 Втр 17:42:17 #99 №238126 
14235793370410.jpg
Такой вопрос - известно, что умножение можно свести к сумме операций сложения (н-р, 5х2 = 5+5). А можно ли деление свести к сумме операций вычитания?
Аноним 10/02/15 Втр 18:10:00 #100 №238129 
>>238126
Если результат деления целое число, то да.
Например, 10/5=10-2-2-2-2
Аноним 10/02/15 Втр 18:12:40 #101 №238130 
>>238125
Но ведь он не математик.
Аноним 10/02/15 Втр 18:18:45 #102 №238132 
>>238126
5x2 = 5 -(-5)
Аноним 10/02/15 Втр 18:19:06 #103 №238134 
>>238126
5x2 = 5 -(-5)
Аноним 10/02/15 Втр 18:23:07 #104 №238135 
>>238130
Зато он ученый и академик.
Аноним 10/02/15 Втр 18:26:04 #105 №238137 
>>238135
Перельман круче. Потому что он хикка.
Аноним 10/02/15 Втр 18:34:28 #106 №238138 
>>238137
Нет, Перельман круче. Потому что он людей лечил.
Аноним 10/02/15 Втр 19:05:09 #107 №238144 
>>238135
Это никак не отменяет, что Перельман-математик жив и поэтому тут нет первокуров которые знали бы о Перельмане до того, как он умер.
И не надо путать "умрет" и "умер". Разумеется тут много первокуров, которые знают о Перельмане до того как он умрет, но нет ни одного, который бы знал о нем до того как он умер.
Аноним 10/02/15 Втр 23:19:17 #108 №238225 
Кто-нибудь может объяснить, с какого хуя в дифференциальной геометрии касательный вектор это оператор дифференцирования?
Аноним 11/02/15 Срд 00:06:32 #109 №238270 
>>238225
Ну, это лишь одно из возможных определений. Можно подойти чисто формально: ввести понятие оператора дифференцирования функций вдоль кривой, потом обнаружить, что множество дифференцирований вдоль всевозможных кривых образует векторное пространство возможна еботня с классами эквивалентности кривых и т.д., не суть, и, соответственно, обозвать касательным вектором v элемент этого пространства. Ничего не объясняя, зачем и почему. Просто потому, что так можно. И задорно кормить хуями людей, пытающихся найти в этом какую-то душевную мотивацию или геометрическую интуицию (любимое развлечение алгебраистов-формалистов).

А можно с самого начала восходить к геометрической интуиции. Она говорит, что вектор по своей глубинной-ололо-сути -- это. в общем-то, так или иначе отсылка к какой-то интуиции направления (направленный отрезок, за одно упоминание которого какая-нибудь тру-нму-блядь будет неистово кормить всех залупами). И если в обычной геометрии поверхностей можно себе касательный вектор тупо нарисовать в приклеенной к данной точке касательной плоскости, вложенной в наблюдаемое пространство, то в дифгеометрии уже хуй так сделаешь: там многообразие определяется чиста-па-сирьезному, своей внутренней структурой безотносительно к вложению в какое-либо объемлющее пр-во, в котором ты бы мог эту касательную плоскость расположить. Соответственно, торчащий из точки касательный отрезок ты нихуя не нарисуешь.

Значит, надо как-то хитровыебанно закодировать понятие направления (="векторности") в точке, да так, чтобы при этом появилось некое векторное типа "касательное" пространство. Очень модно кодировать такие вещи через действие чего-то на что-то. (А чо, вполне себе инвариантненько и во внутренних терминах, без хуйпоймиоткуда торчащих отрезков). А именно: скажем, например, что вектор - это линейный функционал v, действующий на скалярное поле f(x), т.е. сопоставляющий ему некое число v[f], равное, допустим, значению производной от f(x) по направлению какой-нибудь гладкой в данной точке x. И тут внезапно оказывается, что операторы дифференцирования вдоль кривых - весьма годные штуки, образуют векторное пространство, в котором даже (охуеть!) есть кошерный базис. Например, в качестве такового можно взять операторы дифференцирования вдоль линий координатной сетки d/dx^\mu. Ну и назовем тогда касательным вектором элемент этого пространства дифф. операторов.

Более того: разложив его по базису, внезапно обнаружим аналогию с механикой: компоненты разложения по виду сильно напоминают, например, запись компонент вектора скорости частицы d(x^\mu)/dt, кагбе "движущейся" вдоль данной кривой с параметром t. Т.е. выплывает натурально всем известный вектор. Охуевая от таких профитов приложимости всей этой конструкции к задачам механики, допетриваем до её элегантности и простоты уже как части дифф. геометрии. И юзаем.
Аноним 11/02/15 Срд 00:25:09 #110 №238284 
>>238270
Спасибо за объяснение. Но вот есть тогда такие вопросы:
1. Хорошо, вот допустим взяли такое определение касательного вектора. Логично, что тогда всю дальнейшую диффгеому надо бы строить на нём. Но известно, что есть ещё одно, более интуитивное, но громоздкое определение: просто берем кривые в многообразии, проходящие через точку, локально фигачим в R^n,у кривой в R^n берем обычный вектор скорости в образе нашей точки многообразия. У меня бомбит, когда в нашем курсе диффгеомы свободно пользуются и тем, и другим определением. От этого складывается ощущение, что где-то есть обман.
2. Потом можно доказать, что любое гладкое многообразие вкладывается в R^2n. Соответственно, можно уже там взять обычное касательное пространство в R^2n,которое очень хорошо можно представить. Но мы-то строили теорию на том определении. Почему теперь эти два касательных пространства - одно и то же? Нет, ну понятно конечно что у них одинаковая размерность и как векторные пространства они изоморфны, но всё же неясно, какое отношение уже будут иметь операторы дифференцирования к обычным векторам в R^n.
Аноним 11/02/15 Срд 01:18:04 #111 №238290 
>>238284
>У меня бомбит, когда в нашем курсе диффгеомы свободно пользуются и тем, и другим определением.

Я так думаю, что это просто от разных традиций идет. Есть подход как бы "бескоординатный" (функционалы там, алгебра дифференцирований, формы), а есть идущий от Рашевских и прочих классиков тензорного анализа координатный подход, где сразу стартуют с компонентного языка типа компонент вектора скорости. На самом деле подходы вроде как эквивалентные, для общих теорем удобней первый, для вычислений в базисе - второй. Как-то так.

>какое отношение уже будут иметь операторы дифференцирования к обычным векторам в R^n.
Конечно, их можно отождествить по изоморфизму. Подозреваю, что соответствие будет по тем же причинам, по которым оно имеет место в случае касательных векторов к кривым и поверхностям в R^3. Т.е. можно, видимо, всю эту схему с геометрией касаний перенести в R^2n, что там принципиально изменится, кроме увеличения числа компонент векторов?
Аноним 11/02/15 Срд 09:25:52 #112 №238324 
>>237334
Лол. Только сегодня, перелистывая новости, узнал, что за физия на оппике. ОПу плюсик в кармочку.
Аноним 11/02/15 Срд 10:53:47 #113 №238339 
>>238324
ПОКА ОНА НЕ ПОЛУЧИЛА ФИЛДСА О НЕЙ НИКТО НЕ СЛЫШАЛ ПОК ПОК ПОК ПОК ПОК
Аноним 11/02/15 Срд 11:15:17 #114 №238342 
>>238290
А зачем тогда давать па-сирьезному опредление гладкого многообразия, если оно в конце концов всё равно отождествляется с многообразием в Rn ?
Аноним 11/02/15 Срд 11:28:55 #115 №238343 
>>237454
Что за байка?
Аноним 11/02/15 Срд 12:17:09 #116 №238348 
>>237334
> я за творчество, математика не для меня
Проиграл с пидарахи. Творчество без математики, блядь. Охуеть вообще.
Аноним 11/02/15 Срд 14:02:56 #117 №238378 
>>238225
Мехмат, я прав?
Аноним 11/02/15 Срд 14:14:54 #118 №238383 
>>238342
Потому что определение, избегающее лишних сущностей, более просто и экономно, и позволяет не вводить в каждой задаче костыль в виде вложения в R^n.
Аноним 11/02/15 Срд 14:54:11 #119 №238411 
А вот и нмупонтоёб вылез >>238378
Аноним 11/02/15 Срд 15:15:41 #120 №238425 
>>238411
НМУ - это днище вообще, там нихуя не учат, только выёбываются.
Аноним 11/02/15 Срд 18:18:45 #121 №238575 
>>238425
Звучит как тост.
Аноним 11/02/15 Срд 19:59:13 #122 №238585 
Был недавно в НМУ, понравился магазин "Мат. книга". Видел девку в майке с надписью "Я МАТЕМАТИК". Это норма для НМУ?
Аноним 11/02/15 Срд 20:13:30 #123 №238586 
>>238585
>Видел девку ... Это норма для НМУ?
Нет.
Аноним 11/02/15 Срд 20:44:57 #124 №238594 
14236766975090.jpg
Аноним 11/02/15 Срд 21:23:50 #125 №238617 
>>238378
Ну, собственно, переставь 2 слога и получишь ответ.
Аноним 11/02/15 Срд 23:09:00 #126 №238641 
Ребят, душевно реквестирую книжку, обыскал все, будьте добры, кто что может

"Counterexamples in topology" Steen , Seebach
Аноним 11/02/15 Срд 23:23:50 #127 №238645 
>>238594
Надо было на сраке наколоть.
Аноним 11/02/15 Срд 23:27:12 #128 №238648 
>>238641
>обыскал все
За две секунды нашел в LibGen с кучей зеркал.
http://gen.lib.rus.ec/search.php?req=Counterexamples+In+Topology&lg_topic=libgen&open=0&view=simple&phrase=1&column=def
Как можно таких вещей не знать?
Аноним 11/02/15 Срд 23:38:29 #129 №238654 
>>237579
>если с "вышкой" у тебя было все хорошо
У меня было все хорошо с решением задач. Но вот физическую и геометрическую интерпретацию производной я прояснил в голове только месяц назад. Такие дела. Теперь же мне стало интересно упорядочить в голове все основные методы дифференциирования и интегрирования, обогатить свои знания методами статистической математики и т.д.
>Просто бери того же Сивухина
Да ну нахуй, учитывая то, что у меня лежит в бумаге Берклеевский курс физики.
>>237574
Бамп вопросу.

Кстати, лежит в бумаге курс Кудрявцева и курс Ильин-Садовничий-Сендов по мат. анализу. Какому отдать предпочтение? Или достаточно Мышкиса?
Аноним 12/02/15 Чтв 00:05:26 #130 №238661 
>>238648
Огромное спасибо, я еще рад бы записать себе подобные сервисы с книжками, чаю тебе.
Аноним 12/02/15 Чтв 00:16:16 #131 №238664 
>>238594
Все чаще кажется, что я умру либо холостым, либо мучаясь от не сразу замеченной тупости своей спутницы жизни
Аноним 12/02/15 Чтв 00:18:50 #132 №238666 
>>238664
Либо в баттхертах от того, что тян окажется твоей коллегой умнее тебя.
Аноним 12/02/15 Чтв 00:31:24 #133 №238669 
>>238666
Это же лютый вин. Таких не встречал, хотя и не искал совершенно.
Аноним 12/02/15 Чтв 01:10:15 #134 №238675 
>>237334
Как же я бугурчу от таких мразей, для которых математика = арифметика + решение квадратных уравнений.
Сделай с собой что-то плохое, я тебя ненавижу
Аноним 12/02/15 Чтв 01:31:01 #135 №238679 
14236938618790.jpg
>>238675
А если не сделаю?
Аноним 12/02/15 Чтв 01:45:54 #136 №238680 
>>238641
А я реквестирую то, что реквестировал ещё в прошлом треде - книжку с контрпримерами по геометрии и алгебре, аналогичную контрпримерам в анализе. За топологию спасибо.
Аноним 12/02/15 Чтв 12:09:17 #137 №238733 
14237321572640.jpg
>>237578
Не обязательно математика. Как точно подмечено в тексте: "как всегда бывает с чисто умственной деятельностью". Проблема действительно с миром и самой основой жизни. Иррациональность жизни ведет к невозможности "наслаждаться жизнью", при этом трезво мысля. При детальном разборе окружающая нас общественная жизнь становится фарсом, глупостью, скукотой. Что бы наслаждаться жизнью, чувствовать её, то не надо думать, надо именно чувствовать.
Это цена за ту самую радость и удовлетворение. На мой взгляд вполне достойная.
>>237634
Как раз в задротстве и не важно в каком.
>>237671
> твой социальный круг в курсе чем ты занят и поддерживает тебя
Надеюсь такой нам встретятся.
Аноним 12/02/15 Чтв 12:12:08 #138 №238734 
B cтаде 101 коза. Если увести любую козу, то оставшихся можно разделить на два стада по 50 коз в каждом, так что суммарный вес первого стада равен суммарному весу другого стада. Докажите, что все они весят одинаково.
Аноним 12/02/15 Чтв 16:03:31 #139 №238758 
>>238733
>окружающая нас общественная жизнь становится фарсом, глупостью, скукотой
Она и является фарсом, глупостью, скукотой. Чтобы наслаждаться жизнью, имея активный мозг, нужно просто жить иной жизнью.
Аноним 12/02/15 Чтв 16:07:05 #140 №238759 
14237464252780.png
сап математики
сейчас почитал про число Грэма
говорят есть еще больше (естественно используемые в мат. доказательствах)
это какие?
Аноним 12/02/15 Чтв 16:13:46 #141 №238760 
>>238734
Докажем по индукции, что теорема верна для любого нечётного числа коз.
Для трёх коз теорема очевидно верна.
Пусть она верна для 2n+1 коз.
Добавим в стадо две козы. Пусть для этого нового стада выполняется условие. Тогда из предположения индукции легко вывести, что вес двух добавленных коз одинаков.
Выберем из стада одну из добавленных коз. Тогда другая коза попадёт в одно из стад. Оригинальные козы попарно компенсируют друг друга, нескомпенсированной останется только одна коза. Поскольку предположение индукции выполняется, вес оригинальной козы равен весу добавленной козы. Таким образом, все козы в стаде из 2n+3 = 2(n+1) +1 коз весят одинаково.

В том числе и в стаде из 101 козы.
Аноним 12/02/15 Чтв 16:43:26 #142 №238763 
>>238760
Ладно, а что если вес козы — действительное число?
Аноним 12/02/15 Чтв 16:55:59 #143 №238765 
>>238763
А пофиг.
Аноним 12/02/15 Чтв 18:20:21 #144 №238769 
>>238765
Ой, ну конечно, почему-то подумал, что индукция по весу козы. Остаётся один вопрос: эта задача из кружка малого Мехмата МГУ для 5 класса (http://mmmf.msu.ru/archive/20102011/z5/25.html). Неужели пятиклассники должны были использовать индукцию?
Аноним 12/02/15 Чтв 18:22:21 #145 №238770 
>>238769
Почему нет?
Помню на ЛМШ рассказывали про индукцию, когда я был ещё шестиклассником. Все отлично понял. Индукция вообще довольно очевидная вещь
Аноним 12/02/15 Чтв 18:37:48 #146 №238772 
>>238770
А теперь ты где учишься? (Просто интересно.)
Аноним 12/02/15 Чтв 18:44:41 #147 №238774 
>>238769
Да, конечно. Индукция - это как раз уровень четвёртого-пятого класса. Просто у нас образовательная система заточена под олигофренов.
Аноним 12/02/15 Чтв 18:55:53 #148 №238779 
>>238774
Ну, согласен. В моей школе в "физмат-классе" в 10 классе только показали индукцию, и то ничего толком не объясняли (полурока уделили, а потом снова дрочить тригонометрические формулы), и как следствие никто не понял суть (за исключением тех, кто уже знал).
Аноним 12/02/15 Чтв 20:44:22 #149 №238795 
>>238779
>>238774
В демидовиче первый параграф посвящен мат. индукции.
Аноним 12/02/15 Чтв 22:55:10 #150 №238825 
Существует ли FAQ, в котором были бы названы самые хорошие русскоязычные учебники по каждому из разделов математики?
Аноним 12/02/15 Чтв 23:22:00 #151 №238833 
>>238825
на русскому вроде нету, есть гайд с форчана
Аноним 13/02/15 Птн 01:03:09 #152 №238864 
>>238833
Можно линк, о чем речь?
Аноним 13/02/15 Птн 01:10:43 #153 №238865 
Есть ли смысл начинать читать книги на английском по математике для первого курса? Если у меня уровень инглиша довольно низок. Кто-то говорил, что там всего 200 слов.
Аноним 13/02/15 Птн 01:16:04 #154 №238867 
>>238865
>Есть ли смысл начинать читать книги на английском по математике для первого курса

Только если они безальтернативны. Что для уровня матана первого курса, обсосанного дохуя раз в течение десятилетий на любых языках кроме древнегреческого... ну ты понел. Выбирая между продиранием через дебри плохо знакомых тебе слов в ущерб тонкостям смысла и легким скольжением по смыслу в ущерб знанию иностранной лексики, выбирай смысл, если тебе важнее быстрее и качественнее понять математику, блять, а не задрочиться на "ололо-всёрусское-параша".
Аноним 13/02/15 Птн 02:36:49 #155 №238879 
>>238864
https://sites.google.com/site/scienceandmathguide/subjects/mathematics
Аноним 13/02/15 Птн 09:34:58 #156 №238909 
>>238760
Лажа. С чего ты взял, что после убирания двух коз будет выполнено условие для 2n-1?
А суть в том, что у всех коз одинаковый остаток mod 2. Тогда можно поменять веса по правилу x -> x/2 (или x -> (x-1)/2), после чего условие сохранится, а такими операциями все сведется к весу 0 у всех. Случай рациональных сводится натуральным. Случай расширения рациональных к рациональным, т.к. можно выбрать базис в расширении и сравнивать все покоординатно.
Как случай кроме рациональных должны решать пятиклассники без понятия.
Аноним 13/02/15 Птн 10:07:47 #157 №238913 
>>238909
>с чего ты взял
Ещё один не может в метод индукции.
Аноним 13/02/15 Птн 13:13:26 #158 №238959 
x² + y² = N. Найти max(x+y). Без тригонометрии.
Аноним 13/02/15 Птн 14:28:36 #159 №238975 
Добра вам, господа. Не прошу ничего решать, просто укажите название теоремы или метода, способного решить систему из трех уравнений, коэфициэнты и свободные члены известны, а переменных всего три. Но они записаны в виде

a11(x/x) + a11(y/x) + a13(z/x)
a21(x/y) + a21(y/y) + a23(z/y)
a31(x/z) + a32(y/z) + a33(z/z)

Как я понимаю, все три метода Крамера, Гауса и матричный, которые я знаю тут не подходят. Заранее спасибо.
Аноним 13/02/15 Птн 14:40:43 #160 №238979 
>>238975
Уравнение-то где? Ты только левые части указал.
На x, y, я[/z] z домножить не пробовал?
Аноним 13/02/15 Птн 14:41:06 #161 №238980 
>>238979
(опс, обосрался с разметкой)
Аноним 13/02/15 Птн 14:50:55 #162 №238986 
>>238979

Уравнения? Ок. Изначально они выглядели так:

Ar(a)x/(Ar(a)x+Ar(b)y+Ar(c)z)) = ω(a)
Ar(b)y/(Ar(a)x+Ar(b)y+Ar(c)z)) = ω(b)
Ar(c)z/(Ar(a)x+Ar(b)y+Ar(c)z)) = ω(z)

Где Ar 0 относительный атомный вес элемента, а омега - процентное содержание элемента в молекуле. Про домножение не думал, так как если домножить, переменная окажется в свободном члене. Конечно, так можно сделать матрицу из 4 переменных в каждой строке и без свободного члена, но я опять же не знаю как ее решить.
Аноним 13/02/15 Птн 15:22:33 #163 №238992 
>>238986
После домножения у тебя просто получится 3 линейных уравнения с нулевым столбцом свободных членов. Т.е. однородная система линейных уравнений. Она имеет либо только тривиальное решение x=y=z=0 (которое не подходит, т.к. в исходной системе у тебя иксы были в знаменателе), либо (если детерминант матрицы коэфф. при неизвестных равен нулю) имеет бесконечное множество решений, выражающееся через фундам. систему решений.

Так что какая-то странная у тебя система.

http://ru.wikipedia.org/wiki/%D0%A0%D0%B5%D1%88%D0%B5%D0%BD%D0%B8%D0%B5_%D1%81%D0%B8%D1%81%D1%82%D0%B5%D0%BC_%D0%BB%D0%B8%D0%BD%D0%B5%D0%B9%D0%BD%D1%8B%D1%85_%D0%B0%D0%BB%D0%B3%D0%B5%D0%B1%D1%80%D0%B0%D0%B8%D1%87%D0%B5%D1%81%D0%BA%D0%B8%D1%85_%D1%83%D1%80%D0%B0%D0%B2%D0%BD%D0%B5%D0%BD%D0%B8%D0%B9
Аноним 13/02/15 Птн 15:23:43 #164 №238993 
>>238986
>Ar(c)z/(Ar(a)x+Ar(b)y+Ar(c)z)) = ω(z)
Кстати, там вместо ω(z) справа ω(с) должно быть?
Аноним 13/02/15 Птн 15:37:22 #165 №238996 
>>238993
>>238992
Да, там должно быть c, это вообще говоря индекс. Спасибо вам, господа-математики. Судя по всему, искомый ответ - это наименьшые (целые) решения фср. Осталось только понять сам метод. Спасибо еще раз.
Аноним 13/02/15 Птн 16:26:52 #166 №239006 
14238340122720.png
Центр масс треугольника — точка пересечения медиан. Почему тогда пикрелейтед показывает точку Ферма, которая отличается от центроида?
Аноним 13/02/15 Птн 16:51:44 #167 №239013 
>>238867
Я просто думал, что там не так много слов надо знать, а поэтому можно без ущерба для смысла читать на англ, спасибо.
Аноним 13/02/15 Птн 18:23:12 #168 №239033 
>>238959
Произведём замену u = x/sqrt(N), v = y/sqrt(N). Тогда u^2 + v^2 = 1, а значит max(u+v) = sqrt(2) (это можно показать заменой u на sin, v на cos и введением дополнительного угла). При этом x+y = sqrt(N)(u+v), поэтому max(x+y) = sqrt(2N)
Аноним 13/02/15 Птн 20:28:48 #169 №239075 
Линейная алгебра:
1) Беклемишев
2) Ильин, Позняк
3) Гельфанд
Что выбрать?
Аноним 13/02/15 Птн 21:03:46 #170 №239079 
>>239075
Винберг
Аноним 14/02/15 Суб 00:06:47 #171 №239106 
>>239079
А из перечисленных троих?
Аноним 14/02/15 Суб 03:08:27 #172 №239142 
>>239106
Очевидные кострикин-манин.
Аноним 14/02/15 Суб 07:29:51 #173 №239173 
>>239142
Лол.
Аноним 14/02/15 Суб 10:58:45 #174 №239200 
Вбросьте пасту с форчонга, где советуют книги для первого курса. Там еще суммарная стоимость 200-300 баксов и вся программа займет 2.5 года.
Аноним 14/02/15 Суб 13:29:34 #175 №239228 
>>239142
Это только два. Третий-то кто?
inb4 Винберг
Аноним 14/02/15 Суб 13:35:59 #176 №239230 
>>239075
Sheldon Axler
Аноним 14/02/15 Суб 17:25:12 #177 №239296 
>>239033
Там же тебе анон написал, что без тригонометрии странное ограничение, обычно бывает из уст тупых преподов
Ну, можно через условный экстремум по методу Лагранжа.
F(x)=(x+y)-λ(x^2+y^2-N)
∂F/∂x=1-2xλ =0 =>λ=1/2x;
∂F/∂y=1-2yλ =0 => 1-2y/(2x)=0 => y=x;
∂F/∂λ=-(x^2+y^2-N)=0 => x^2+x^2=N => x=y=sqrt(N/2);
Тогда x+y=2sqrt(N/2)=sqrt(2N).

Тут еще надо бы проверить, что это действительно максимум, т.е. посмотреть матрицу 2-х произодных, но это уже мелочи.

>>239075
Если физик - то Ильин-Позняк норм.
Аноним 14/02/15 Суб 18:14:24 #178 №239304 
>>239296
Физфак МГУ? Или откуда такое говноедство?
Аноним 14/02/15 Суб 18:34:15 #179 №239306 
>>239304
>Физфак МГУ
>говноедство
Пшел вон, пёс
Аноним 14/02/15 Суб 18:43:49 #180 №239308 
>>239306
НИ ЭНМЭУ ЗНАЧИТ ГАВНИЦО)))
Аноним 14/02/15 Суб 21:45:54 #181 №239370 
>>239296
Никому не советую читать линал Ильина Позняка. Серьезно. Даже физикам.
Аноним 15/02/15 Вск 00:02:23 #182 №239395 
>>238114
В каком разделе изучаются свойства кривых, такие как, замкнутость, непрерывность, деление точек плоскости на два класса? Слышал что топология за это отвечает.
Аноним 15/02/15 Вск 01:48:25 #183 №239413 
>>239370
Why?
Аноним 15/02/15 Вск 10:03:42 #184 №239430 
>>239075
Гельфанд, потом Кострикин-Манин
Аноним 15/02/15 Вск 11:32:33 #185 №239456 
>>237603
А что такое Гротендик?
Аноним 15/02/15 Вск 11:45:23 #186 №239466 
>>239456
Он ебет твою мамку.
Аноним 15/02/15 Вск 12:09:53 #187 №239471 
Чем обмазаться чтобы знать математику начиная где-то с 9-го класса, в условиях нехватки времени етц (т.е. желательно одна книга, с пояснениями для быдлогуманитария)?
Аноним 15/02/15 Вск 12:22:38 #188 №239473 
>>239430
Почему не Винберг -> Кострикин-Манин? Или Кострикин (введение в алгебру) -> Кострикин-Манин?
Аноним 15/02/15 Вск 14:16:43 #189 №239487 
>>237334
Математика не наука, а язык описания окружающего мира. Не понимаешь математику - не понимаешь абсолютно ничего.
sageАноним 15/02/15 Вск 14:37:56 #190 №239493 
>>239487
Двачаю вам мсье
Аноним 15/02/15 Вск 14:48:44 #191 №239500 
>>239487
Я не понимаю диффуры, зато понимаю что твоя мамка шлюха, а сам ты нищий задрот.
Я товар за 2 доллара покупаю в китае, за 20 продаю тут и спокойно живу на эти десять процентов.
сейчас кто-то влепит хуйню не осознавая что это анекдот
Аноним 15/02/15 Вск 14:51:56 #192 №239502 
>>239500
Без диффуров этот товар бы попросту к тебе не доехал.
Аноним 15/02/15 Вск 15:18:05 #193 №239509 
>>239473
В Винберге не только линейка, ты же просил линейку.

Кострикин (введение в алгебру), я с нее долго плевался. Уж очень меня раздражал стиль подачи материала, тот же Винберг гораздо лучше этого поделия.

>>239471
Зачем тебе знать математику?
Аноним 15/02/15 Вск 16:16:42 #194 №239518 
>>239413
Потому что хуйня, которое создает не понимание предмета, а инвалидность, препятствующую понимаю в полной мере.

А вообще, идите нахуй, биомусор, кричащие про нму и вшэ. Линал - не набор формальных фактов, как учат в ильине-позняке, например.
Линал - это алгебра векторных пространств, гибкая, подвижная, визуальная.
Не хочу чтобы перваки травмировались этим говном по незнанию. Вы же не математики, блять, математик не может советовать вредный учебник, если сам не инвалид.
Аноним 15/02/15 Вск 16:51:18 #195 №239520 
>>239518
>Линал - не набор формальных фактов, как учат в ильине-позняке, например

Ладно, ок, морфизмы-функторы-дуальные пространства-коммутативные диаграммы - модули над полем-идеалы-хуялы ---- вот это всё ты называешь линейной алгеброй? Ок. В Ильине этого нет.

Зато есть линейные пространства (метрические, гильбертовы, евклидовы и комплексные), лин. операторы (вкл. самосопряженные), системы линейных уравнений и методы их решения, спектральные задачи, матричная алгебра, квадратичные/билинейные формы, поверхности 2 порядка, тензоры, основы групп (включая группу Лоренца) и их представлений. Для физиков, например, - вполне норм (именно поэтому на физфаке это стандартный учебник).
Аноним 15/02/15 Вск 17:06:50 #196 №239521 
>>239520
Ладно, ок, алгебра-геометрия-физика-химия-биология-хуелогия... вот это всё ты называешь наукой? Ок. В учебнике науки этого нет.
Зато есть млекопитающие (рукокрылые, парнокопытные, китообразные и ондатры), устройство аккордеона, таблицы интегральных преобразований и умножения, число 8, спутники Урана (вкл. Нептуна), описание химических реакций и задач небесной механики, зайцеобразные, основы мандарина, животные и их рога. Для меня, например, - вполне норм (именно поэтому в моей шараге это стандартный учебник науки).
Аноним 15/02/15 Вск 17:15:23 #197 №239522 
>>239502
Думаешь, диффуры товар из Китая везут? А вот хуй тебе, это мои рабы с телегой. А ты продолжай сношаться со своими диффурами.
Аноним 15/02/15 Вск 17:18:16 #198 №239523 
>>239521
Подъебал типа)
Если бы в Ильине было всё свалено в кучу - тогда бы аналогия прошла. Только там не свалено в кучу. Изложение вполне последовательное, самозамкнутое, практически с нуля, с определениями-леммами-теоремами. Да, это изложение не всего линала, и не для pure-математиков. Однако изложен вполне солидный кусок, имеющий значение для физиков в их прикладных задачах и являющийся базой для мат. аппарата квантмеха и теории относительности.
inb4 физика без функторов и модулей над полем - не физика
Аноним 15/02/15 Вск 17:19:51 #199 №239524 
>>239523
Именно поэтому учившиеся по нему люди думают, что теория самосопряжённых операторов - часть теории линейных операторов. Хороший учобник, чо.
Аноним 15/02/15 Вск 17:55:13 #200 №239529 
>>239524
В основе - тот же линал, только расширенный до функановских изъебтств в бесконечномерных пространствах.
Аноним 15/02/15 Вск 19:30:36 #201 №239542 
>>239520
Нет, я сейчас называю линалом то, что им называют в МГУ. И как раз в этом смысле Ильин-Позняк плохой учебник. В качестве учебника по алгебре вообще он не плохой, а крайне плохой.

> модули над полем
> Ок. В Ильине этого нет.
Значение знаешь?
Аноним 15/02/15 Вск 19:46:11 #202 №239549 
>>239542
>Значение знаешь?
Да. И именно в такой формулировке там этого нет. А есть просто 8 аксиом.
Аноним 15/02/15 Вск 20:11:11 #203 №239559 
>>239542
>то, что им называют в МГУ
Смотря на каком факультете.

Имхо, спор аналогичен спору "что такое теормех". Для мехматянтовцев - всякая арнольдоподобная поебень с дифурами, пучками и аттракторами. Для инженегров - расчет балок и шарикоподшипников. Для физиков - уравнения Лагранжа и Гамильтона-Якоби, частицы в центральных полях, рассеяние, вариационка и т.п. по образцу ландафшица.
Аноним 15/02/15 Вск 20:26:33 #204 №239564 
>>238913
Ну, давай. Объясни, почему если при выкидывании любой козы можно поделить пополам, чтобы было равенство, то и при выкидывании любых ТРЕХ так можно сделать (ведь ты убрал две произвольные козы и требуешь, чтобы условие сохранялось).
Это ты мудак, который не знает, что в индукции вообще нельзя использовать слово "добавить" только убрать. Давай ты мне еще докажешь, что в любом связном графе отличном от дерева есть тройной цикл (для трех верно, при добавлении вершины цикл никуда не денется).
Ты хоть понимаешь, в чем проблема?
Аноним 15/02/15 Вск 21:42:47 #205 №239580 
>>238760
>Для трёх коз теорема очевидно верна
База индукции, ОК
>Пусть она верна для 2n+1 коз
Индуктивное предположение, ОК
>Добавим в стадо две козы
ОК, стало 2n+3
>Пусть для этого нового стада выполняется условие.
Что значит "пусть выполнятся"?? Для 2n+3 как раз это и нужно доказать!

Аноним 15/02/15 Вск 21:48:15 #206 №239583 
>>239580
Стоп. Претензия снимается. "выполняется условие" =/= выполнятся утверждение теоремы.
Аноним 16/02/15 Пнд 14:32:53 #207 №239674 
>>238760
Ебать ты графендик)))0)))

Слушаете мою задачку, берем некоторое натуральное эн, если оно четное, то делить пополам, если нечетное, то множим на 3 и прибавляем единицу, получим новое число, к которому применим ту же самую операцию. Надо доказать, что рано или поздно для любого изначально взятого числа мы получим в конце концов единицу
Аноним 16/02/15 Пнд 14:40:03 #208 №239676 
>>239674
че тут доказывать-то) сам попробуй и убедись)
Аноним 16/02/15 Пнд 15:45:16 #209 №239682 
>>239674
Назовём числа, применение к которым описанного тобой алгоритма даёт единицу, сводимыми.
2 сводимое, очевидно.
Пусть все числа, меньше n+1, сводимые.
Докажем, что n+1 сводимое.
Если n+1 чётное, то в результате деления пополам получим число меньшее, чем n+1, а оно сводимое, значит и n+1 сводимое.
Если n+1 нечётное, то оно представимо в виде 2k+1. Применяем к нему вторую операцию. 3(2k+1) + 1 = 6k + 4 = 2(3k+2). Получили чётное число, делим пополам, получаем 3k+2.
Если k чётное, то k=2p. Тогда 3k+2 = 6p + 2 = 2(3p + 1). Делим пополам, получаем 3p + 1, оно меньше чем 4p+1 и поэтому сводимое.
Если k нечётное, то k=2p+1. Тогда 3k+2 = 3(2p+1) + 1 = 6p + 4 = 2(3p + 2). Делим пополам, получаем 3p + 2. Так как 3p + 2 < 2k+1 = 2(2p+1) + 1 = 4p + 3, оно сводимое.
Поэтому все натуральные числа сводимые.
Аноним 16/02/15 Пнд 15:52:40 #210 №239683 
>>239682
Ты обосрался со случаем k=2p+1. Хоть бы проверил разок перед отправкой. Даже сомнения не закралось, почему независимо от чётности k, 3k+2 у тебя получается чётным.
Аноним 16/02/15 Пнд 16:21:15 #211 №239684 
>>239674
Первокур, плиз
https://ru.wikipedia.org/wiki/%D0%93%D0%B8%D0%BF%D0%BE%D1%82%D0%B5%D0%B7%D0%B0_%D0%9A%D0%BE%D0%BB%D0%BB%D0%B0%D1%82%D1%86%D0%B0
Аноним 16/02/15 Пнд 16:28:08 #212 №239685 
>>239684
Мне эта задача напомнила теорему Гудстейна, думал, что >>239674 -хуй нашел другую формулировку и решил тут всех затраллить. А вон оно как оказалось.
Аноним 16/02/15 Пнд 20:37:33 #213 №239720 
14241082532350.png
14241082532361.png
Посаны, как это понимать?
Аноним 16/02/15 Пнд 21:29:58 #214 №239730 
>>239720
Любой
Аноним 16/02/15 Пнд 21:33:53 #215 №239733 
>>239730
В смысле любое из утверждений не выполняется.
Аноним 16/02/15 Пнд 21:35:28 #216 №239734 
>>239730
Я вот не могу понять, как английские математики друг друга понимают, если у них зачастую слова имеют противоположные в рамках терминологии значения? В русском такого не замечал, а с английским у меня не особенно хорошо, так что в голове не укладывается.
Аноним 16/02/15 Пнд 22:16:27 #217 №239741 
>>239734
Приводи примеры или хуй простой. Любой из и каждый из это синонимы.
Аноним 16/02/15 Пнд 22:20:50 #218 №239746 
>>239741
Я имею в виду два варианта перевода: "один из двух" и "и тот и другой". Это уже разные вещи.
Аноним 16/02/15 Пнд 22:23:21 #219 №239748 
>>239746
В этом случае одно из.
Аноним 16/02/15 Пнд 22:26:55 #220 №239750 
>>239748
Ну и как я это должен понять?
Аноним 16/02/15 Пнд 23:19:57 #221 №239755 
>>239750
Используй ум, если он у тебя есть.
Аноним 16/02/15 Пнд 23:43:26 #222 №239756 
>>239746
В данном случае "один из двух" означает не "только один, а другой не обязательно" а имеет смысл "ты можешь выбрать один из двух, любой, и оно будет неверным". Что синонимично тому, что ложны должны быть одновременно оба утверждения.
Аноним 17/02/15 Втр 02:25:47 #223 №239768 
14241291474630.jpg
Аноним 17/02/15 Втр 02:27:48 #224 №239769 
>>239768
Странно, что нашелся лишь один дворник с калькулятором.
Аноним 17/02/15 Втр 19:13:55 #225 №239891 
>>237334
Котишки, есть такая функция, которая, допустим, непрерывна на [a, b], но на любом интервале из этого отрезка она немонотонна.
Аноним 17/02/15 Втр 20:01:51 #226 №239905 
14241925112930.jpg
>>239891
Это утверждение, а не вопрос.
Аноним 17/02/15 Втр 20:55:39 #227 №239926 
>>239905
Забыл знак вопроса поставить. Так есть или нет?
Аноним 17/02/15 Втр 21:00:48 #228 №239929 
>>239926
Есть.
Гелбаум, Олмстед. Контрпримеры в анализе.
С. 42.

http://vk.com/doc18010776_252155374?hash=59682aa5c461742e66&dl=6cd698ac1b7422c437
Аноним 17/02/15 Втр 21:10:04 #229 №239934 
>>239905
Предел sin nx при n стремящемся к бесконечности?
Аноним 17/02/15 Втр 21:10:21 #230 №239936 
>>239929
Какая книженция охуенная, помню мне ее советовали где-то, но я забыл про нее, спасибо.
Аноним 17/02/15 Втр 21:15:21 #231 №239938 
>>239934
Что? Если n натуральное, то она не непрерывна, если вещественное, то она будет монотонна.
Аноним 17/02/15 Втр 22:28:02 #232 №239954 
Братцы, как распробовать анализ? Алгебра и дискретка, ТВ, Мат Логика нормально идет, но от всякого функана дикая тошнота и желание дропнуть.
Спасибо
Аноним 17/02/15 Втр 22:29:59 #233 №239955 
>>239954
Алсо, на данный момент осилил Фихтенгольцовские основы мат анализа
Аноним 18/02/15 Срд 01:07:54 #234 №239981 
>>239955
>Фихте
Блеванул. Решай листки.
Аноним 18/02/15 Срд 01:14:20 #235 №239984 
>>239981
Нму проститутка закукарекала
Аноним 18/02/15 Срд 01:27:56 #236 №239987 
>>239981
приличные люди листками жопу вытирают
листки для сектантов
Аноним 18/02/15 Срд 10:12:10 #237 №240006 
14242435305580.png
Посоны, объясните умственно отсталому что это за элементарная формула откуда она и как называется.
Аноним 18/02/15 Срд 10:15:48 #238 №240007 
>>240006
Ассоциативность умножения в поле
Аноним 18/02/15 Срд 10:33:53 #239 №240008 
14242448337710.png
>>240007
Ассоциативность умножения как говорит загнивающая пикрелейтед.
Могу задать вопрос проще, ребенок в школе изучает арифметику и попросил ему помочь.
Там элементарная задача с процентами и решается она элементарно, а вот объяснить ребенку почему так получается не могу.
Взял записал эту хуйню в виде формулы, но так нихуя и не понял.
Я в школе в математику вообще не вкуривал.
Анон спасай мою личинку, не дай ей вырасти таким же долбаебом как папаша.
Аноним 18/02/15 Срд 10:44:25 #240 №240012 
Задача такова:
Найти 115% от числа 2860.
Решение: 2860/100115=3289 (нашел один процент от суммы и умножил на необходимое кол-во)
Но со школы я помню что могу сделать проще:
2860
1,15=3289 что равносильно 115/100*2860 т.е. по этой >>240006 формуле.
А вот почему так можно не могу ни объяснить ни сам понять.
Аноним 18/02/15 Срд 10:47:04 #241 №240013 
Блядь вакаба распознала две звездочки как тег и въебала курсив, там где начинается и заканчивается курсив там должен быть символ умножения
Аноним 18/02/15 Срд 14:18:10 #242 №240031 
14242582900300.gif
>>240008
В глаза ебусь коммутативность конечно же
Аноним 18/02/15 Срд 14:23:21 #243 №240032 
>>240031
Ассоциативность здесь >>240006. Потому что в первом случае мы сначала x умножаем на 1/y, а потом всё на z, а во втором случае z умножаем на 1/y, а потом всё на x.
Аноним 18/02/15 Срд 14:24:59 #244 №240034 
>>240008
Главное не допускай такого http://janatem.livejournal.com/79065.html
Аноним 18/02/15 Срд 14:31:45 #245 №240036 
>>240034
Ухх, хорошо-то как!
Аноним 18/02/15 Срд 14:33:57 #246 №240037 
>>240032
Да, так, просто порой путаюсь в этом
Аноним 18/02/15 Срд 14:59:57 #247 №240041 
>>240034
Пиздец, конечно. Помню еще в школе никак не мог понять, почему говорят, что кубического корня из -8 не существует. Оказалось, так делают "для упрощения".
Аноним 18/02/15 Срд 15:14:18 #248 №240042 
>>240041
лол кубический корень из -8, это -2
Аноним 18/02/15 Срд 15:26:40 #249 №240046 
14242624009150.png
>>240042
Но даже в хороших учебниках пишут, что его нет (Гельфанд и Шень, например). И мне в школе так говорили.
Аноним 18/02/15 Срд 15:39:31 #250 №240051 
>>240046
Неудивительно, авторы как настоящие профессионалы своего дела, просто стараются избегать данных скользких моментов, которые сложно разрешить не используя, к примеру комплексный анализ.
Аноним 18/02/15 Срд 15:42:26 #251 №240052 
>>240046
Более того на пике, ничего не говорится о том что его нету
Аноним 18/02/15 Срд 17:00:40 #252 №240056 
14242680406430.png
Существует ли подмножество P множества всех функций R -> R такое, что выполняется пикрелейтед (вместо числа будет функция)?
Аноним 18/02/15 Срд 18:34:44 #253 №240060 
>>240056
Возьми значение функции в какой-то фиксированной точке, лол.
Аноним 18/02/15 Срд 18:39:47 #254 №240061 
>>240060
В фиксированной точке a значение функции может быть равно 0, и тогда f(a) и -f(a) одновременно принадлежат или не принадлежат P, чего не может быть.
Аноним 18/02/15 Срд 19:02:19 #255 №240062 
>>240032
Ассоциативность это x⋅(1/y ⋅ z) = (x⋅1/y) ⋅ z в силу которой можно получить выражение xz/y но не zx/y. Его можно получить в силу именно коммутативности:
x⋅1/y ⋅ z = x⋅ z ⋅ 1/y = xz/y = x/y ⋅z
x⋅1/y ⋅ z = x⋅ z ⋅ 1/y = z⋅ x ⋅ 1/y = zx/y = z/y ⋅x




Аноним 18/02/15 Срд 22:33:32 #256 №240077 
>>240056
Я хз, придумал тут набросок доказательства - только оно, по-моему, с говнецом неконструктивное.

Рассмотрим множество F ненулевых функций. Каждой такой функции f(x) можно всегда сопоставить функцию g(x)=-f(x). Скажем, что две функции эквивалентны, если отличаются максимум знаком. Тогда все множество F разбивается на непересекающиеся классы эквивалентности, каждый из которых состоит ровно из двух элементов (пары противоположных по знаку функций).

Рассмотрим множество Q всех таких классов эквивалентности. Используя аксиому выбора, выберем из каждого класса q∈Q по одному элементу (функции) λq(x), сформировав тем самым некоторое "выбранное" семейство функций P={λq(x) для всех q∈Q}.

Так как любая функция f(x) попадает в какой-то класс эквивалентности, то либо она была выбрана в процессе формирования P (т.е. f(x)∈P), либо не она (а тогда вместо нее в P попала противоположная ей функция: -f(x)∈P).

Нулевая функция f(x)=0, как и требуется, стоит особняком.
Аноним 18/02/15 Срд 23:15:55 #257 №240079 
>>240077
Норм вроде. А я пытался доказать, что нет такого P, но ниче не получилось.
Аноним 18/02/15 Срд 23:45:08 #258 №240082 
>>239075
Чем плох/хорош курс Беклемишева (вкупе с "Дополнительными главами по линейной алгебре" того же автора)?
науч. сотрудник-инженегр-кун
Аноним 19/02/15 Чтв 00:04:51 #259 №240083 
>>240082
Норм. курс (особенно последнее издание), все четко без лишних заебов и наворотов. Для физиков и инженеров вполне годно. (Математики, естественно, будут плеваться и выражать претензию на то, что лишь они понимают, каким должен быть истинный линал.)
Аноним 19/02/15 Чтв 00:15:09 #260 №240084 
>>240083
>особенно последнее издание
У меня есть книги из 80-х, думаю разницы с современными переизданиями никакой особо и нет. Тем более, что "дополнительные главы" с тех пор не переиздавались (а частичный материал из них впихнули в новые переиздания).
Аноним 19/02/15 Чтв 02:21:43 #261 №240094 
14243017031070.jpg
14243017031181.png
Сап матаны, нужно ваше мнение, пилю слегка морозную.
Битард 25 лвл, за ручку держался один раз в жизни в пионер лагере еще ребенком, конечно же листва, через пять лет кастану свой первый фаербол. В школе учился на одни тройки, просто потому что было не интересно и как то похуй. В математике был ПОЛНЫЙ ноль, учителям было на меня просто похуй и они рисовали мне тройки закрыв глаза.
Полное незнание математики всегда нагревало мой пукан, и в какой то мере у меня появился комплекс неполноценности по этому поводу. Как только вижу формулу или дробь ГРОБ ГРОБ КЛАДБИЩЕ ПИДОР КРОВЬ КИШКИ РАСПИДОРАСИЛО и мозг просто выключается как будто.
Дело в том, что так как я имею достаточно широкий кругозор то я всегда понимал что математика нужна везде куда ни сунься. Но всеми силами избегал ее изучения, так как считал, что мне ее никогда не осилить.
Моему терпению пришел конец недавно, когда нужно было высчитать параллактическое смешение и я охуел от формул.
Мое незнание математики не дает мне саморазвиваться дальше.
И вот я решил что я начинаю изучать математику, это в 25 лет блядь, потому что это главным образом повысит мою самооценку и поможет мне избавится от этого ебучего комплекса.
Перерыв кучу учебников и посмотрев учебные планы школьной математики за начальные классы, понял что начинать мне нужно с АРИФМЕТИКИ БЛЯДЬ, стыдно пиздец как.
В итоге остановился на "Справочнике по элементарной математике" Выгодского.
Сейчас я почти закончил раздел арифметики и скоро перейду к элементарной алгебре.
В ходе изучения арифметики испытывал трудности с пониманием вроде элементарных вещей и бывало по часу думал над какой-нибудь элементарной хуйней, но в конце концов все таки самостоятельно вкуривал.
ВНИМАНИЕ ВОПРОС:
Говорит ли то что я испытывал трудности в изучении арифметики о том что я долбаеб, ну в смысле о низкой степени интеллекта? Или это просто психологическая травма с детства? Или это нормально когда человек ни разу в жизни не пользовавшийся мозгом вдруг при первом его использовании испытывает трудности?
Или вся арифметика должна пониматься взрослым человеком элементарно?
Есть ли у меня шанс нормально шарить в математике?
Извини за стену текста анон, просто накипело.
Содержание части "арифметика" из учебника Выгодского прилагаю.
Аноним 19/02/15 Чтв 02:31:44 #262 №240095 
>>240056
Эээ, множество функций, заданное явно например, конечным перечислением так, чтобы это выполнялось? Либо я не понимаю проблемы, либо вербитодауны в конец ебанулись.
Аноним 19/02/15 Чтв 02:53:44 #263 №240097 
>>240094
Что в твоем понимании >нормально шарить в математике
?
Аноним 19/02/15 Чтв 02:58:19 #264 №240098 
>>239734
Мне понятен смысл всегда. Ты пытаешься применять переводы из словаря, вместо того, чтобы понять смысл слова и как оно используется. Рекомендую использовать английский толковый словарь (Webster, MacMillan etc.).

Я не математик.
Аноним 19/02/15 Чтв 03:22:00 #265 №240101 
>>240097
Шарить в моем понимании значит не просто применять готовые формулы как обезьянка камень, а понимать почему эти формулы работают.
Аноним 19/02/15 Чтв 09:14:34 #266 №240129 
>>240094
Начни не со справочника, а с учебников.
http://rutracker.org/forum/viewtopic.php?t=3294592
http://rutracker.org/forum/viewtopic.php?t=3294635
Потом прочти по возростающей
http://rutracker.org/forum/viewtopic.php?t=3967454

Если шаришь в английском, параллельно смотри:
http://rutracker.org/forum/viewtopic.php?t=2003248
https://www.youtube.com/playlist?list=PL301908982CBFE20D
Если не шаришь:
https://www.youtube.com/playlist?list=PLxGo9dxQkqWBHGfHY5rypgSDaXKCT8EZJ
Аноним 19/02/15 Чтв 09:45:37 #267 №240132 
Кстати, раз уж речь зашла об элементарной прикладной математике: если ли какие-то книги, посвященные построению математических моделей по условиям задачи? Знаю, у Савченко есть "Задачи по физике с анализом их решения", но это не совсем то.
Аноним 19/02/15 Чтв 11:17:04 #268 №240147 
14243338240430.jpg
>>240129
От души анон. Скачал учебники, подписался на канал.
Еще вопрос:
В каком порядке изучать математику?
У Выгодского как то мне кажется более логично:
1. Арифметика
2. Алгебра
3. Геометрия
4. Тригонометрия
5. Графики, функции

В остальных же учебниках все как то вперемешку и это меня дезориентирует.
Хотя в школе насколько я помню алгебру и геометрию преподавали параллельно. (из-за того что алгебру и геометрию преподавали отдельно я до недавнего времени думал что это разные предметы, а оказалось это просто два раздела математики)
Так вот, если я буду изучать математику в последовательности справочника Выгодского не будет ли это долбаебизмом?
Или и геометрию и алгебру нужно изучать параллельно?
Аноним 19/02/15 Чтв 12:49:19 #269 №240149 
>>240147
Анон, зачем тебе? Ты хочешь угореть по математике как таковой, или интересуют связанные с ней прикладные области?

Аноним 19/02/15 Чтв 13:05:18 #270 №240150 
>>240149
>Анон, зачем тебе?
Написано здесь >>240094
Мне нужен полный школьный курс математики, в большем объеме она мне навряд ли понадобится.
Вкратце, для дальнейшего расширения кругозора и решения задач в естественно-научной среде.
Аноним 19/02/15 Чтв 13:30:55 #271 №240155 
>>240147
Во-первых, с функциями тебе придется работать уже при изучении элементарной алгебры, например. Тригонометрия тебе если и нужна, то только совсем базовые вещи: определение триг. функций в прямоугольном треугольнике, дальнейшее определение для числового аргумента, несколько основных тождеств и понимание сути аркфункций (то, что традиционно проходят в школе в 10-11 классах по тригонометрии — хуйня, состоящая чуть более чем полностью из тупого задрачивания десятков формул и преобразований однотипных выражений). Итак, арифметика тебе, безусловно, нужна полностью (плоть до понятия простых чисел, делимости, сравнения по модулю, алгоритма Евклида, метода индукции). На алгебру не смотри как на тупое преобразование громадных формул, как это часто бывает в школе: главное для тебя понять, каким образом алгебра вырастает на фундаменте в виде арифметики, как над одночленами и многочленами вводят операции сложения/умножения/..., что есть корень, как найти корни квадратного трёхчлена (не тупое вызубривание формулы дискриминанта, а понимание теоремы Виета, хотя бы), как разложить на множители. В геометрии главное видеть, как из некоего набора правил (аксиом), которые мы сами условились ввести, следуют теоремы (тут я бы посоветовал учебник Атанасян 7-9 кл.). Параллельно можешь смотреть, например, этот курс http://www.youtube.com/watch?v=rQJMT9nbFhk
Аноним 19/02/15 Чтв 13:36:37 #272 №240156 
>>240150
>решения задач в естественно-научной среде.
Тогда тебе нужно делать упор на анализ и линейную алгебру. Брось школьную арифметику складывать дроби, я надеюсь, ты умеешь?, купи лучше калькулятор, а если купишь инженерный, то можешь и на тригонометрию с логарифмами хуй положить.
Сперва Шеня Алгебру ебани:
http://www.mccme.ru/free-books/shen/gelfand-shen-algebra.pdf
Потом Анализ Зорича попробуй:
http://rutracker.org/forum/viewtopic.php?t=1055058
По геометрии можешь полистать учебник Погорелова 7-11 кл. для общего развития.
Аноним 19/02/15 Чтв 13:40:19 #273 №240157 
>>240150
Справочники Выгодского по элементарной и по высшей математике это конечно хорошо, но для того чтобы вот прям с нуля достигнуть "расширения кругозора и решения задач в естественно-научной среде" их будет маловато.
Аноним 19/02/15 Чтв 13:40:55 #274 №240158 
>>240155
>понятия простых чисел, делимости, сравнения по модулю, алгоритма Евклида, метода индукции
Ну и нахуя ему всё это? Он олимпиадные задачки задрачивать не собирается, насколько я понял?
Аноним 19/02/15 Чтв 13:43:20 #275 №240160 
>>240158
При чём тут олимпиадные задачи? Понимание математики в принципе невозможно, если даже не знаешь перечисленных мною вещей.
Аноним 19/02/15 Чтв 13:58:26 #276 №240164 
>>240160
Знать в общих чертах достаточно, прочитать и забыть, если ты не олимпиадник.
Всё нужное всплывет в алгебре при дальнейшем изучении, причем в нормальном виде, индукцию можно будет доказать, в арифметике же она смотрится как костыль.
Аноним 19/02/15 Чтв 14:04:40 #277 №240167 
>>240164
>индукцию можно будет доказать, в арифметике же она смотрится как костыль
Абсолютно не согласен. С чего это она как костыль? Во-первых, в арифметике можно ввести индукцию как аксиому, либо ввести аксиому о существовании наименьшего элемента и доказать индукцию как теорему. И в арифметике индукция появляется очень явно, без неё вообще никуда.
>Всё нужное всплывет в алгебре
Всплывает, но это как сказать "Учи с первого класса матан, всё нужное из школьной алгебры и арифметики там всплывает". Основываться надо на элементарных вещах, а уже дальше простые числа перетекают в неприводимые многочлены, делимость чисел в делимость многочленов, НОД чисел в нутыпонял...
Аноним 19/02/15 Чтв 14:26:58 #278 №240169 
>>240095
> one and only one
А, ну да, надо было лечь спать и утром уже подумать.
Аноним 19/02/15 Чтв 14:32:21 #279 №240171 
>>240167
>>240167
>ввести индукцию как аксиому
Так делается в школьных учебниках, и понимания которого так желает вопрошающий анон это не добавляет, полный днэн индукции достигается только алгеброй, ящитаю. Во всяком случае, думаю, не стоит сильно заморачиваться на элементарной теории чисел, если собираешься решать естественно-научной задачи.
>делимость чисел в делимость многочленов
Это важно для решения внутриматематических проблем (разрешимость уравнений, возможность построений). Сомневаюсь, что прикладникам так уж необходимо знать признаки делимости многочленов.
Аноним 19/02/15 Чтв 14:49:21 #280 №240173 
>>240155
Спасибо анон. Суть ясна.
Но ты так и не ответил как изучать алгебру и геометрию, последовательно или параллельно?
Скачал учебник Атанасян, действительно годнота, видно по содержанию, да и иллюстрации доходчивые.
На тот канал подписался посмотрю на днях, там четыре таких лекции по 2 часа каждая насколько я понял.
Аноним 19/02/15 Чтв 15:02:44 #281 №240178 
>>240156
Понял, сначала буду разбираться с элементарной алгеброй (погуглил и насколько я понял линейная алгебра пока для меня за гранью понимания).
>Брось школьную арифметику складывать дроби, я надеюсь, ты умеешь?
Ты не поверишь, только в 25 лет этому научился, серьезно, стыдно пиздец. Я раньше мог складывать лишь десятичные дроби, но не простые.
Блядь, алгебра Шеня ОХУЕННАЯ ГОДНОТА.
С нее кстати и начну, уже полистал наспех и понял что расписано прям так как мне надо (т.е. как для долбаебов) и свойства арифметических операций расписаны очень просто, я этого нигде не мог найти. Блядь охуенный учебник, спасибо.
А вот второй учебник открыл и что то лес какой то. У Атанасян все систематизировано, как мне и нравится, с нее и начну.
А что ты скажешь о том как изучать геометрию и алгебру последовательно или параллельно?
Аноним 19/02/15 Чтв 15:24:06 #282 №240185 
>>240178
Наверни немного алгебры сперва, как только разберешься с базовыми арифметическим и алгебраическим понятиями (сложение дробей, возведение в степень, многочлены, простенькие уравнения), тогда можно смело браться за геометрию. В общем-то, большего базиса для изучения элементарной геометрии и не требуется, как мне кажется.
Аноним 19/02/15 Чтв 16:19:11 #283 №240197 
>>240094
1) Алгебра Гелфанд Шень
2) Какая-нибудь небольшая книжка о том, как доказывать теоремы.
3)
http://www.mccme.ru/free-books/yaschenko/v08book-08.pdf
http://www.mccme.ru/free-books/yaschenko/v08book-09.pdf
4) вузовские учебники, сам разберешься какие
Аноним 19/02/15 Чтв 16:22:59 #284 №240199 
>>240158
Блять, если ты не знаешь метода индукции, как ты вообще теоремы собрался доказывать? А если ты не собираешься доказывать теоремы, то нахуй ты нужен, если все, что ты умеешь, комп делает за милисекунду?
Аноним 19/02/15 Чтв 16:45:31 #285 №240203 
>>240199
Лучше почитай диалог полностью.
В любом нормальном учебнике алгебры на первых страницах есть теорема о трансфинитной индукции, которая, понятное дело, в дальнейшем используется для доказательства многих теорем.
Но какие теоремы ты, чепушило, собрался доказывать девятиклассникам урезанной школьной мат. индукцией, я не знаю. Школота использует индукцию, чтобы доказывать тождества, неравенства и решать всякую олимпиадную хуиту, но анону, который просил совета, это не нужно.
Аноним 19/02/15 Чтв 16:55:51 #286 №240205 
>>240199
посчитай мне континуальный интеграл на компьютере мамкин гротендик
Аноним 19/02/15 Чтв 17:04:44 #287 №240207 
>>240203
> тождества, неравенства
> олимпиадная хуита
ясно
Аноним 19/02/15 Чтв 17:29:21 #288 №240212 
>>240205
Посчитал.
Аноним 19/02/15 Чтв 18:12:05 #289 №240219 
>>240203
>теорема о трансфинитной индукции
В Винберге нет, в Кострикине тоже и в Городенцеве, про Куроша вообще молчу. Даже у Шафаревича нет.

Мат. индукцией доказывают дохуя вещей в матлоге, немного в матане и, вроде, в алгебре/ангеме применял пару раз.
Аноним 19/02/15 Чтв 18:48:10 #290 №240226 
>>240197
2 пункт — "Простейшие примеры мат. доказательств", МЦНМО.
3 пункт считаю излишним для первого знакомства с математикой (тогда уж лучше "Что такое математика?" включить, но и она на уровень выше, мне кажется)
>>240171
Я понимаю твои мысли, но мне кажется, что ОП не столько собирается на самом деле решать прикладные задачи, сколько просто немножко разобраться с математикой. Я правильно понимаю, ОП? Ты же не будешь после изучения заниматься какими-то прикладными областями математики? А для понимания предмета математики как раз таки линал какой-нибудь особо не нужен (слишком сложно), а вот арифметика в самый раз.
Аноним 19/02/15 Чтв 20:36:22 #291 №240252 
>>240156
Ну пиздец, ты бы ещё ему с Теории Множеств Бурбаки посоветовал бы начать. А чо, там же с самых основ все расписано
Аноним 19/02/15 Чтв 21:17:21 #292 №240257 
>>240219
В Винберге действительно нет, ну хуй знает, может потому что там идёт уклон в линал. У Алуффи и Хангерфорда есть.
>>240252
А что? Ну Шень ему понравился. Зорича пусть попробует после, не зайдет, ну и хуй с ним.
Погорелов вообще хуйня.
Аноним 19/02/15 Чтв 22:49:40 #293 №240281 
>>240147
>В каком порядке изучать математику?
Просто читай учебники, от 5-го класса к 11-ому. Собственно, учебники для того и пишутся, чтобы просвещать людей с нулевыми знаниями. Тем более, сама структура учебников предполагает краткое повторение в следующих частях (за старшие классы) ранее изученного материала, или рассмотрение ранее изученного материала с новых точек зрения, что позволяет закрепить знания в уме.
Цель? Теоретическая цель очень проста: после того как ты прочитаешь все эти учебники, Алгебру Шеня и т.д., ты должен будешь закрепить все прочтением этой книги
http://rutracker.org/forum/viewtopic.php?t=2466586
Если ты все поймешь - можешь двигаться дальше. Например, начать решать задачки или еще что-то. На геометрию можешь забить хуй, как по мне. То есть, того, что написано в учебнике Сканави или каких-то справочниках, вполне достаточно.

>>240129-кун
Аноним 20/02/15 Птн 00:38:47 #294 №240304 
>>237334
Возникло желание проникнуть в тыл к мамке опа
>Математика после 6 класса - сплошной флуд
Сам учусь на физикоматематическом факультете, в 10 классе, и поверь, то, что мы проходим - невероятно легко. Я заглянул в учебный план по Мише Вербицкому, и что же я вижу - проходимые нами(в школе) темы - лютая xyNTa, по сравнению с тем, что действительно было бы интересно. Учебный курс же в общеобразовательных классах заслуживает многократных беспрерывных фэйспалмов на протяжении нескольких часов, ибо их полугодовую программу можно освоить за неделю-две "рядовому ученику". Поэтому сопли беспричинны и неоправданны.
Реактивный говномет и ссанно-тряпкометатель сняты с предохранителя и приведены в боевую готовность.
Аноним 20/02/15 Птн 00:56:07 #295 №240308 
>>240304
>Я заглянул в учебный план по Мише Вербицкому
Блять.
Аноним 20/02/15 Птн 01:00:53 #296 №240310 
>>240304
Ну и нахуя ты с пастой говоришь? Очевидно же, что ОП-пост туда прилепили только ради лулзов.
Аноним 20/02/15 Птн 02:19:39 #297 №240317 
14243879790560.jpg
Собственно есть неоднородное диф уравнение с кратными корнями на фоточке. Решая методом эйлера встает вопрос будет ли общее решение таковым х=4с1-1с2, у=-1с1+(1/4)с2
Аноним 20/02/15 Птн 02:22:36 #298 №240319 
>>240317
Подъезжая к сией станцыи и глядя на природу в окно, у меня слетела шляпа.
Не думал, что когда-нибудь увижу такое на бордах.
Аноним 20/02/15 Птн 02:31:20 #299 №240321 
>>240317
Поехавший
Аноним 20/02/15 Птн 04:20:16 #300 №240326 
>>240156
Проклинаю того уебка, который мне посоветовал Алгебру Шеня. До чего гнусный долбоеб, этот Шень, он поместил в конце глаз совершенно невыносимую олимпиадную хуйню, блядь, да я нахуй на выносе множителей за скобку завис на месяц нахуй с циклическими мать его перестановками.
Аноним 20/02/15 Птн 04:23:35 #301 №240327 
>>240326
Нуок, в следующий раз сказку "Колобок" посоветую.
Аноним 20/02/15 Птн 05:35:12 #302 №240328 
14243997121640.png
>>240327
ахуенно полезные задачки. Смекалочка должна быть из определенного места.
Аноним 20/02/15 Птн 07:06:10 #303 №240331 
>>240328
Дык а чо - все примитивненько так. Надо просто посмотреть, при каких комбинациях типа a+-b равно нулю все обращается в ноль. В первой получится типа (x-y)(y-z)(z-x). В 157 вынестся (a+b+c), 158 (a+b)(b+c)(c+a). В 159 вынесется (a-b) и по циклу
Аноним 20/02/15 Птн 13:45:32 #304 №240421 
>>240326
> олимпиадную хуйню
Если бы эти задачи были олимпиадными, то любой человек с iq > 70 был бы олимпиадником.
Аноним 20/02/15 Птн 13:51:40 #305 №240423 
>>240331
А 156 нельзя разложить, потому что дискриминант отрицательный. Мне кажется, >>240326 просто тупой. Хотя пасту смешную сгенерировал.
Аноним 20/02/15 Птн 14:16:20 #306 №240426 
>>240423
И что с того, что дискриминант отрицательный? Давай ты мне так ещё докажешь, что x^4+x^2+1 не раскладывается.
Аноним 20/02/15 Птн 14:26:30 #307 №240429 
>>240426
Ой
Аноним 20/02/15 Птн 14:35:27 #308 №240432 
>>240429
Ебать, а как тогда разложить?
http://www.wolframalpha.com/input/?i=x%5E10+%2B+x%5E5+%2B+1+%3D+0
Аноним 20/02/15 Птн 14:41:17 #309 №240434 
14244324778100.png
>>240432
Охуеть, реально гниль олимпиадная. Я не понимаю, какую ценную математическую идею содержит такое упражнение.
Аноним 20/02/15 Птн 14:42:48 #310 №240435 
>>240432
Я тебе открою секрет.
http://www.wolframalpha.com/input/?i=factor+x^10+%2B+x^5+%2B+1
Перемножаешь скобки в правой части разложения. НИЧЕГО НЕ СОКРАЩАЕШЬ. А потом хуяк такой взмахиваешь хуем волшебной палочкой и возвещаешь - да будет добавлено и вычтено хуй знает сколько слагаемых, чтоб потом фсё взяло да вынеслось за скобки!
Вот потому такие задачки - и есть полное говно
Аноним 20/02/15 Птн 14:57:25 #311 №240439 
>>240434
Оно содержит в себе только одну идею: проиллюстрировать, что такое неприводимые разложения над полями. Задрачиваться над их вычислениями вручную (когда есть алгоритм Кронекера) - все равно что перемножать пятизначные числа в уме (когда есть калькулятор) или брать интегральчики под картофанчик (когда есть вольфрам альфа).
Аноним 20/02/15 Птн 14:57:33 #312 №240440 
>>240432
На самом деле можно там разложить самому. Увидеть что x^2+x+1 делит довольно просто, зная его корни (Это корни третьей степени из единицы, подставляешь их в x^10+x^5+1 и получаешь 0). Далее делишь, получаешь ту хрень восьмой степени. Рассматриваешь её редукцию по модулю 2, видишь что у неё нет корней. Значит исходный неприводим.
Аноним 20/02/15 Птн 15:47:58 #313 №240451 
>>240440
Ой, я хуйню снёс. То что у редукции нет корней ещё ничего не значит.
Аноним 20/02/15 Птн 15:53:47 #314 №240453 
>>240434
Ну отдать должное Шеню, там есть задачи наводящие на такое решение. Однако, угадывая многочлен x^2+x+1 можно потратить достаточно времени.
Аноним 20/02/15 Птн 16:06:48 #315 №240455 
>>240326
Прорешал всего Шеня классе в 11. Ничего сложного там не встретил. Зато навсегда запомнил вывод некоторых ебанных формул, которые теперь повсеместно встречаются в университетском курсе.
Та же тригонометрия гораздо изъебистей и доконает окончательно в курсе матана хитрыми преобразованиями.
Аноним 20/02/15 Птн 16:11:16 #316 №240456 
>>240455
>вывод некоторых ебанных формул, которые теперь повсеместно встречаются в университетском курсе.
Хуй знает о чем ты. Неужели о (x+y)^n?
Аноним 20/02/15 Птн 17:14:25 #317 №240468 
Я бы просто разложил на (a^5 - exp(2 i pi/3))(a^5 - exp(-2i pi /3 )). Потом посчитать корни пятой степени из комплексной экспоненты и сгруппировать по сопряженным парам. В одной паре действительно вылезет (a² +a+1), в остальных парах коэффициенты будут иррациональные.
Аноним 20/02/15 Птн 17:19:44 #318 №240469 
14244419846510.jpg
Други
На пикче изображена задачка. Есть общее решение волнового уравнения. Есть начальное условие. Есть джва граничных условия: одно нормальное, а другое заданное НЕХ, включающей произвольную функцию f.
Внимание, вопрос: как получить частное решения волнового уравнения? Я так понимаю, что надо решить НЕХ на границе, а потом действовать стандартным методом? И если это так, то как решить НЕХ? Я просто немного нуб в дифурчиках, и мб это уравнение тоже какое-то всем известное, кроме меня? Пробовал решать его вольфрамом, он послал меня нахуй. Такие дела.
Аноним 20/02/15 Птн 18:40:11 #319 №240501 
>>240469
Чёта не выглядит так, чтобы ответ не зависел от выбора функции f.
Аноним 20/02/15 Птн 19:52:53 #320 №240536 
>>240501
Да-да, я понимаю, что ответ будет от нее зависеть. Но, может, его можно как-то выразить через f и ее производные/первообразные?
Аноним 20/02/15 Птн 20:13:45 #321 №240538 
>>240536
Я имею ввиду что как раз способ, с помощью которого ты должен вообще подходить к решению, зависит от функции f. В дифуравнениях а тем более в урчп лел часто такая ситуация, что даже для чуть чуть разных краевых/начальных условий получаются совершенно разные подходы, часто привлекающие методы из разных далёких областей математики, по каждому частному случаю пишутся многостраничные справочники. Не вижу причины, по которой здесь должно быть сильно легче. Ведь самое первое, что приходит на ум - это взять общий вид решения и подставить его в граничное условие. Ну и что? при этом получим уравнение с произвольной функцией f. Задаваться вопросом, какие у него решения это всё равно что спрашивать какие решения у уравнения dx/dt=f(x,t) для произвольного вида f.
Аноним 20/02/15 Птн 20:22:53 #322 №240539 
Как MathML в Firefox пофиксить, чтобы он нормально отображал формулы не только в одинарных долларах ($formula$), но и в двойных ($$formula$$)?
Аноним 20/02/15 Птн 20:25:30 #323 №240540 
>>240538
Хм, и что же тогда делать? Может, можно применить какие-нибудь асимптотические методы?
А ещё, эти уравнения взялись не просто так, а из одного нетрадиционного лагранжиана. Мне сказали, что применив теорему нетер к нему и найдя сохраняющиеся величины, мне будет проще решить эту систему. Но равенства теоремы нетер свелись к самим этим уравнениям. Это вообще норма?
Аноним 20/02/15 Птн 21:00:05 #324 №240542 
Некоторые тезисы:
1) Считать можно и на калькуляторе. Знать таблицу умножения или многочисленные языковые правилы нахуй не надо. Надо лишь уметь занести это в калькулятор.
2) Все обучение в школе полная хуйня и построено на ошибочной идее, что знание всей этой "лексики", называемой в школе математикой, полезно в жизни и помогает развить математическое мышление.
3) Самая полезная часть математики в школе - геометрия, учит пространственному мышлению. Хотя 99% посвящено обучению бесполезной математической нотации, так что польза такая себе.
Аноним 20/02/15 Птн 21:04:24 #325 №240543 
>>240542
Тезисы малограмотного школьника с претензией.
Аноним 20/02/15 Птн 21:11:29 #326 №240544 
>>240543
У меня неполное высшее, так что мимо.
Аноним 20/02/15 Птн 21:13:35 #327 №240545 
>>240544
Лалд.

Та и ваще. Усе, чему в школе учуть - полная чушь. Закон Божий надо учить, единственное учение, которое чего-то стоит.
Аноним 20/02/15 Птн 21:19:21 #328 №240547 
14244563613960.png
>>240542
Плавали - знаем 800
Аноним 20/02/15 Птн 21:26:27 #329 №240552 
>>240544
> Подразумевая что ты не являешься малограмотным школьником с неполным высшим
Аноним 20/02/15 Птн 21:26:44 #330 №240553 
>>240547
Хуясе, как ты меня узнал то? Ты думаешь школьники вкладывают в деление с остатком какой-то сакральный смысл? Они просто делают то, чему их научили. Зачем - никто до конца обучения не знает, смысловое продолжение этого материала будет только в институте.

>>240545
Кроме клоунады аргументов не найдется? Я просто искренне не понимаю, где может пригодится владение языком счета. Это бесполезно и в современной науке, и в современной жизни. Бесполезно вообще. Концепций в этом раз-два-три и все они просто могут быть объяснены геометрически
Аноним 20/02/15 Птн 21:46:24 #331 №240556 
>>240553
> где может пригодится владение языком счета
Считать, молодо зелено.
Аноним 20/02/15 Птн 21:48:19 #332 №240558 
Дан угол и точка М внутри него. Проведите через точку М прямую так, чтобы она отсекала от угла треугольник наименьшей площади.
Аноним 20/02/15 Птн 21:49:00 #333 №240559 
>>240556
>2015
>считать в уме
Ну ясно.
Аноним 20/02/15 Птн 21:52:31 #334 №240561 
>>240558
Ну элементарно же, ну.
Аноним 20/02/15 Птн 21:53:43 #335 №240563 
>>240561
Равнобедренный?
Аноним 20/02/15 Птн 21:58:25 #336 №240565 
>>240563
Сам ты равнобедренный.
Аноним 20/02/15 Птн 21:59:36 #337 №240566 
>>240565
Провести перпендикуляр к стороне, которая дальше, и продолжить его?
Аноним 20/02/15 Птн 22:01:19 #338 №240568 
>>240566
Может, вместо тупого брут-форса ты попробуешь в начале подумать и как-то обосновать свою точку зрения?
Аноним 20/02/15 Птн 22:06:09 #339 №240569 
>>240568
Это не брутфорс, я подумал. Вариант с равносторонним предложил просто как некий вариант симметрии, которая часто имеет место в экстремальных задачах. Потом подумал и понял, что лучше сократить отсекаемый отрезок на луче, который дальше от точки (потому что тогда удлиним отрезок на другом луче не настолько сильно). А изначально просто пытаюсь уменьшить произведение ab, из формулы площади sinAa*b/2.
Может быть, я просто спать хочу... дай наводку.
Аноним 20/02/15 Птн 22:06:57 #340 №240571 
>>240559
Ясно.
Аноним 20/02/15 Птн 22:10:02 #341 №240573 
>>240569
Ну чо. Пишешь такой функцию площади в зависимости от угла наклона прямой, дифференцируешь, получаешь решение. Вуаля. И водовки не забудь навернуть
Аноним 20/02/15 Птн 22:12:56 #342 №240574 
>>240573
Ну это понятно, но задача-то для школьников, вроде, хочу как-то красиво решить.
Аноним 20/02/15 Птн 22:13:12 #343 №240575 
>>240569
Да этот даун тралит, очевидно же, что он сам не знает ответа.
Аноним 20/02/15 Птн 22:13:49 #344 №240576 
>>240574
а с каких пор школьники не знают производных?
Аноним 20/02/15 Птн 22:16:29 #345 №240577 
>>240576
Ну ладно, как тут чисто из школьных знаний о производной решить? Я так понимаю, угол между нашей прямой и биссектрисой брать, а что дальше?
Аноним 20/02/15 Птн 22:24:50 #346 №240578 
>>240577
> между стороной
фикс
Аноним 20/02/15 Птн 22:26:32 #347 №240579 
>>240469
бамп вопросу
Аноним 20/02/15 Птн 22:31:40 #348 №240582 
>>240569
>просто пытаюсь уменьшить произведение ab
Единственная годная мысль. Еще можно вспомнить, что стороны в треугольнике связаны.
Аноним 20/02/15 Птн 22:36:50 #349 №240583 
>>240582
Ну и что, решение-то какое?
Аноним 20/02/15 Птн 22:40:22 #350 №240585 
>>240583
$\frac{1}{k} = \frac{x_0}{y_0} - \frac{2}{tan(\alpha)}$
Про обозначения сам догадаешься, надеюсь
Аноним 20/02/15 Птн 22:45:33 #351 №240587 
>>240585
Нет
Аноним 20/02/15 Птн 22:47:02 #352 №240588 
>>240434>>240435
Какой же ты тупой уебок. Это про циклотомические многочлены, что достаточно важная тема в теории Галуа и для теории чисел полезно. И если понимать, что такое корни из единицы, то это все, блядь, очевидно.
Аноним 20/02/15 Птн 22:50:46 #353 №240589 
>>240588
>>240588
-->240439
Аноним 20/02/15 Птн 22:51:03 #354 №240590 
>>240587
http://latex.codecogs.com/gif.latex?$$\frac{1}{k}%20=%20\frac{x_0}{y_0}%20-%20\frac{2}{tan(\alpha)}$$
Очевидно же, что самая удобная система координат - когда вершина угла лежит в нуле, а ось икс - одна из сторон угла. alpha - очевидно, угол, x_0 и y_0 - координаты точки М в этой системе, а к - коэффициент наклона прямой, при которой площадь отсекаемого треугольника минимальна
Аноним 20/02/15 Птн 22:59:19 #355 №240591 
>>240590
Ну может тебе и очевидно такое решение, а мне, школьнику без знаний аналгема, не очевидно, как к такому прийти.
Аноним 20/02/15 Птн 23:36:51 #356 №240594 
>>240591
Аналгем - вымышленный предмет, им нельзя владеть, там нет никакой теории, никаких знаний, только пара приемчиков, которые дают еще в школе.
Аноним 20/02/15 Птн 23:58:27 #357 №240604 
>>240542
>Все обучение в школе полная хуйня и построено на ошибочной идее, что знание всей этой "лексики", называемой в школе математикой, полезно в жизни и помогает развить математическое мышление.
Проблема не математики, а нашей системы. Нету мотивации из самой школьной скамьи. Нету популяризации науки и связанных с нею профессий. Профессия "математик" слишком абстрактна и неясна для школьного ума, он не видит в ней выгоды и будущего. В то время как "чиновник" (которая в большинстве свое состоит из гумманитариев, олигархов, спотсменов и гэбни), "бизнесмен", "топ-менеджер" вполне конкретны и показательны. Это только если с самого детства не мылить лыжи на съеб из сраной СНГшки, но мало кто осознает данную необходимость в столь нежном возрасте.
Аноним 21/02/15 Суб 00:23:05 #358 №240607 
14244673858550.jpg
>>240604
>Профессия "математик" слишком абстрактна и неясна для школьного ума
>для школьного ума
Что абстрактного-то? Пикрелейтед - профессия "математик".
Аноним 21/02/15 Суб 00:28:37 #359 №240608 
>>240607
Это препод какой-то, а "математик" может легко быть и связанным с вполне коммерческой отраслью. Например, составлять планы для микросхем (теория графов в чистом виде), алгоритмы для анализа данных, анализировать финансовые системы и т.д.
Аноним 21/02/15 Суб 00:36:59 #360 №240609 
>>240607
Наверное, только 0,001 % теперешних школьников страстно желает занять место данного профессора в будущем.
>>240608
>составлять планы для микросхем (теория графов в чистом виде), алгоритмы для анализа данных, анализировать финансовые системы и т.д.
Слишком неконкретно. У нас кто-то выпускает микросхемы? Это распространенная профессия с массой предприятий в каждом городе?
>алгоритмы для анализа данных, анализировать финансовые системы
Мой знакомый, закончивший магистратуру по системному анализу, в итоге пошел в говно-кодеры. Такие дела.
Аноним 21/02/15 Суб 00:56:37 #361 №240612 
>>240609
>Это распространенная профессия с массой предприятий в каждом городе?
Нет, но так с любой профессией. Хочешь быть хорошим экономистом — тебе наверняка придется работать в крупном городе с соответствующими твоему профилю предприятиями, с манагерами абсолютно так же. Вот и с математиками: можешь работать быдлокодером или учителем у себя в селе, либо переехать в более-менее крупный город и работать каким-нибудь аналитиком/??? (при условии, что ты хороший математик, конечно).
>Мой знакомый, закончивший магистратуру по системному анализу, в итоге пошел в говно-кодеры
Так же и с другими профессиями. Мой знакомый, закончивший магистратуру по менеджменту, в итоге пошёл делать бигмаки. Тут дело в профессионализме, конкурентоспособности, везении.
Аноним 21/02/15 Суб 01:08:50 #362 №240614 
>>240609
>>240612
Один мой знакомый получил диплом НМУ и теперь работает гей-шлюхой, сосет хуи за деньги.
Аноним 21/02/15 Суб 07:27:35 #363 №240634 
>>240226
>Ты же не будешь после изучения заниматься какими-то прикладными областями математики?
Да, математика ради математики это не для меня.
>>240281
Спасибо анончики, добра.
Аноним 21/02/15 Суб 12:48:31 #364 №240694 
1/a + 1/b + 1/c = 1/(a+b+c). Доказать, что a+b или b+c или a+c = 0.
Аноним 21/02/15 Суб 12:49:41 #365 №240695 
>>240589
И что ты этим хотел сказать? Что >>240439 тоже нихуя не понимает и думает, что раз он не видит смысла, то его нет? Ну да. Так и есть. Как же у этих школьников бомбанет, когда они до приличных учебников дойдут, где будут еще меньше времени уделять на то, чтобы разжевывать все.
Аноним 21/02/15 Суб 13:11:56 #366 №240697 
14245135164170.jpg
>>240694
Пикрелейтед.
>тоже нихуя не понимает и думает, что раз он не видит смысла, то его нет
Иллюстративный смысл к теории полей и колец многочленов - есть (о чем и было сказано), смысла в задротском пафосе умения решать такие задачки вручную - ровно столько же, сколько в картофанчиках (о чем и было сказано). Жопой не читай, да.
Аноним 21/02/15 Суб 13:14:25 #367 №240698 
>>240697
Блин, точно, спасибо. Хотел ведь ещё на множители числитель разложить, но подумал, что ничего хорошего. Почему я должен был догадаться, что эта сумма равна (a+b)(b+c)(a+c)?
Аноним 21/02/15 Суб 13:22:25 #368 №240701 
>>240698
Только эвристически, с опытом задротства на такое. В условии все симметрично относительно a,b,c. Плюс то, что обычно выводы типа "a+b или b+c или a+c = 0" появляются как следствие того, что их произведения равны 0. Значит, где-то эта шняга должна была выползти, а кроме как в числителе - больше неоткуда.
Аноним 21/02/15 Суб 13:42:59 #369 №240705 
>>240588
> теория галуа
> теория чисел
Так бы и сказал сразу, что ты говноед.
Аноним 21/02/15 Суб 13:56:31 #370 №240706 
>>240705
А что не говноедство? Спектралки? Топосы Гротендика?
Аноним 21/02/15 Суб 14:09:51 #371 №240707 
>>240697
>Иллюстративный смысл к теории полей и колец многочленов
Блядь. Я же тебе сказал, что это нихуя не иллюстративный пример, а имеющий прямое отношение к теории Галуа и теории чисел. А также, что никакого задротства тут нет. Любому человеку, который знает про корни из единицы и что-то про них понимает, эта задача очевидна.
Так что это ты жопой не читай. Везде им картофанчик видится.
Аноним 21/02/15 Суб 14:17:05 #372 №240708 
>>240698>>240701
Потому что это многочлен третьей степени, который зануляется на трех плоскостях вида a-b=0 (это из изначального условия очевидно). Значит, его уравнение это в точности произведение линейных.
И никакого задротства.
Аноним 21/02/15 Суб 14:17:53 #373 №240709 
>>240708
Тьфу. Только a+b=0, конечно.
Аноним 21/02/15 Суб 14:46:07 #374 №240714 
Нужна книжка которая научит меня раскладывать полиномы от одной, двух и трех переменных на несводимые.
Аноним 21/02/15 Суб 14:52:02 #375 №240715 
>>240708
>который зануляется на трех плоскостях вида a+b=0
Ну да, тоже как вариант. Хотя это очевидно более-менее опытному, у новичка для быстрой генерации мозгом такого вывода не хватит бэкграунда.

Аноним 21/02/15 Суб 18:01:10 #376 №240737 
>>240714
Метода Кронекера недостаточно?
Аноним 21/02/15 Суб 18:31:31 #377 №240744 
>>240469
бамп вопросу
ответьте, пожалуйста. кто шарит. Может, можно применить какие-нибудь асимптотические методы?
Аноним 21/02/15 Суб 18:32:41 #378 №240745 
1) Как определить категорию без теории множеств???

2) Как можно определить понятие натурального числа через более элементарные понятия? Но только без скрытой рекурсии.
Аноним 21/02/15 Суб 18:36:49 #379 №240746 
>>240608

Это не математик, а прикладник. Дно ёбаное.
Аноним 21/02/15 Суб 18:47:59 #380 №240750 
>>240707
>прямое отношение
В чем заключается это отношение? В мотивировке, чтобы не ебаться с этим такими кустарными методами? Типа попробуй сначала палочкой потыкать, а потом мы тебе мультитул дадим.
Аноним 21/02/15 Суб 18:52:49 #381 №240752 
>Как определить категорию без теории множеств?

Удваиваю вопрос.
Аноним 21/02/15 Суб 18:54:33 #382 №240753 
>>240752

Ты из haskell-треда?
Аноним 21/02/15 Суб 18:54:51 #383 №240754 
>>240745
1) А ты можешь определить категорию с помощью теории множеств?
2) Как ординалы, например.
Аноним 21/02/15 Суб 19:02:29 #384 №240755 
>>240753
Да, пошёл с тобой посмотреть что тут господа математики ответят, так как самому любопытно.
Аноним 21/02/15 Суб 19:02:29 #385 №240756 
>>240754

1) Все виденные мною определения использовали теоретико-множественный язык.
2) Это как раз пример скрытой рекурсии. Берут готовый натуральный ряд и говорят, что является этим ординалом 0 (кажется) порядка.
Аноним 21/02/15 Суб 19:09:39 #386 №240757 
>>240756
>Берут готовый натуральный ряд
Интересно, а откуда у тебя готовый натуральный ряд взялся?
Аноним 21/02/15 Суб 19:14:25 #387 №240758 
>>240756
Тебя слово "множество" смущает в определении, что ли? Ну замени его на "совокупность".
Аноним 21/02/15 Суб 19:16:57 #388 №240760 
14245354174910.jpg
>>240756
>1) Все виденные мною определения использовали теоретико-множественный язык.
Ты знаешь какую-то другую математическую нотацию? Иллюстрация категорий на примере множеств просто самая доступная.

>2) Это как раз пример скрытой рекурсии. Берут готовый натуральный ряд и говорят, что является этим ординалом 0 (кажется) порядка.
>пример скрытой рекурсии
О, шизик, извини, не признал тебя.

Скрытая рекурсия у здоровых людей называется аксиома. А построить формальную систему без аксиом нельзя.
Аноним 21/02/15 Суб 19:24:07 #389 №240761 
>>240758
Там ещё используется термин класс из теории множеств. Его на что заменить?
Аноним 21/02/15 Суб 19:26:30 #390 №240762 
>>240758

Это позиция страуса прячущего голову в песок. Как не назови это всё равно множество.
Аноним 21/02/15 Суб 19:34:14 #391 №240763 
14245364545630.png
>>240761
А в моем определении даже понятие множества не используется.
Аноним 21/02/15 Суб 19:35:50 #392 №240764 
>>240761
>Там ещё используется термин класс из теории множеств
При этом он не является множеством.
Аноним 21/02/15 Суб 19:35:56 #393 №240765 
>>240745
Вот что нашёл:
http://mathoverflow.net/questions/8731/categorical-foundations-without-set-theory
Аноним 21/02/15 Суб 19:39:46 #394 №240767 
>>240763

Это же Голдблатт. А совокупность и есть множество.
Аноним 21/02/15 Суб 19:43:36 #395 №240768 
>>240767
Нет, совокупность не множество, поэтому и класс не множество, но формулировка там странная, в большинстве учебниках пишут что класс объектов и МНОЖЕСТВО морфизмов.
Аноним 21/02/15 Суб 21:01:36 #396 №240780 
>>240760

> Ты знаешь какую-то другую математическую нотацию? Иллюстрация категорий на примере множеств просто самая доступная.

Я не про примеры категорий, а про само определение в общем случае.

> О, шизик, извини, не признал тебя. Скрытая рекурсия у здоровых людей называется аксиома. А построить формальную систему без аксиом нельзя.

Это вообще разные понятия. Несводимая (по разным соображениям) рекурсия может стать основанием ввести аксиому. Но в этом и есть вопрос, является ли натуральный ряд исходным понятием, или может быть сведем к более простым.

А просто скрытая рекурсия может быть из-за того, что лень заморачиваться с определениями, тем более если книжка не по основаниям математики. 99% вообще в наивной канторовской теории множеств работают.

>>240765

Пробежался, из того что смог понять не нашел ответа. Кто-то упоминает что можно, но не поясняет как и ссылок не даёт.

>>240768

Ну это уже бесполезный спор о терминах. В обычном понимании все три термина означают одно и тоже, классы как супермножества некоторые вводят как попытку избавиться от противоречий, но я этот трюк не понимаю. С тем же успехом можно просто самим себе запреитить все эти известные парадоксы и сидеть в одних множествах. Проблему-то они всё равно не решают.
Аноним 21/02/15 Суб 22:13:07 #397 №240791 
>>240780
>но я этот трюк не понимаю
Есть хорошая иллюстрация того, зачем нужно понятие класса.
Пусть совокупность всех ординалов Г - множество, тогда согласно аксиоме паувер-сета существует множество 2^Г, которое в силу теоремы Кантора имеет мощность больше Г. А значит есть ординал, не вошедший в Г. Получили противоречие. Отсюда понятно, что совокупность всех ординалов не может быть множеством. Следовательно нужно дополнительное понятие. Вот примерно из таких соображений и возникает класс.
Что здесь непонятного?
>является ли натуральный ряд исходным понятием, или может быть сведем к более простым.
Может быть сведен к более простым - к понятию транзитивного множества. Но если мы хотим мыслить натуральный ряд как одно целое, то без аксиомы не обойтись.
Аноним 21/02/15 Суб 22:21:25 #398 №240793 
>>240780
>Ну это уже бесполезный спор о терминах. В обычном понимании все три термина означают одно и тоже, классы как супермножества некоторые вводят как попытку избавиться от противоречий, но я этот трюк не понимаю. С тем же успехом можно просто самим себе запреитить все эти известные парадоксы и сидеть в одних множествах. Проблему-то они всё равно не решают.
Тут каждое предложение неправильное.
Аноним 21/02/15 Суб 22:24:39 #399 №240794 
>>240791
>Может быть сведен к более простым - к понятию транзитивного множества.
Это же то же самое.
Аноним 21/02/15 Суб 22:28:28 #400 №240796 
>>240793

Тут последнее предложение неправильное.
Аноним 21/02/15 Суб 22:36:48 #401 №240799 
>>240794
А что может быть проще транзитивного множества? Только множество, которое вообще никак не определяется. Всё - мы пришли к основам.
Аноним 21/02/15 Суб 22:39:57 #402 №240802 
>>240780
>а про само определение в общем случае.
Ну тогда ты очевидно мудак, потому что в определении нет множеств.

>Но в этом и есть вопрос, является ли натуральный ряд исходным понятием, или может быть сведем к более простым.
Тебе дали равнозначное понятие, которое может быть сведено к более простым. Правда это нахуй не нужно, но все же.
Аноним 21/02/15 Суб 23:17:35 #403 №240816 
>>240799
Во-первых множество ОПРЕДЕЛЯЕТСЯ целой кучей аксиом. Во-вторых СУЩЕСТВОВАНИЕ натурального ряда (бесконечности) в них входит, то есть это нечто очень фундаментальное.
Аноним 21/02/15 Суб 23:23:19 #404 №240818 
>>239891
Есть, ты её как раз определил.
Аноним 21/02/15 Суб 23:25:49 #405 №240821 
>>240818
Неужели ты не понимаешь, что не всё, что можно произнести, существует?
Аноним 21/02/15 Суб 23:36:15 #406 №240822 
>>240816
>множество ОПРЕДЕЛЯЕТСЯ целой кучей аксиом
Ну определи множество тогда.
Кучи аксиом нужны для образования новых множеств из уже имеющихся.
Аноним 21/02/15 Суб 23:37:21 #407 №240823 
>>240802

> Ну тогда ты очевидно мудак, потому что в определении нет множеств.

Докажи или пиздабол.

> Тебе дали равнозначное понятие, которое может быть сведено к более простым. Правда это нахуй не нужно, но все же.

Какое понятие? Ты только написал херню про аксиомы и всё.

Аноним 21/02/15 Суб 23:54:36 #408 №240831 
>>240823
>Докажи или пиздабол.
Метакатегории, например.

>Какое понятие? Ты только написал херню про аксиомы и всё.
Если ты мудак и не можешь понять самые основы, что с тобой говорить? Пацаны уже топосы гротендика ебашат, а ты все не можешь понять, что такое класс объектов.
Аноним 22/02/15 Вск 00:02:29 #409 №240833 
На полке, переплётами наружу, стоят рядом друг с другом два тома Пушкина. Толщина страниц в каждом томе -- 2 см, а каждой обложки -- 2 мм. Книжный червь сидел на первой странице первого тома и прогрыз (по кратчайшему пути) до последней страницы второго. Какое расстояние он прогрыз?
Аноним 22/02/15 Вск 00:02:58 #410 №240834 
На полке, переплётами наружу, стоят рядом друг с другом два тома Пушкина. Толщина страниц в каждом томе -- 2 см, а каждой обложки -- 2 мм. Книжный червь сидел на первой странице первого тома и прогрыз (по кратчайшему пути) до последней страницы второго. Какое расстояние он прогрыз?
Аноним 22/02/15 Вск 00:07:24 #411 №240836 
14245528442710.jpg
>>240834
Говно какое-то.
Аноним 22/02/15 Вск 00:07:50 #412 №240837 
>>240834
4 мм, если первый том левее второго. Арнольдовский баян (наверняка не он сам придумал).
Аноним 22/02/15 Вск 00:09:56 #413 №240838 
>>240822
Вот грубо:
1. Два множества равны, если они содержат те же элементы.
2. Существует пустое множество.
3. Для любых двух вещей существует множество, единственными элементами которого они являются.
4. Для любого множества множеств существует объединение.
5. Существует натуральный ряд.
6. Аксиома подстановки с вариантами.
7. Для любого множества существует множество подмножеств.
8. Аксиома выбора.
9. Аксиома регулярности.
Для образования новых множеств нужна только часть аксиом. Все они нужны, чтобы прояснить («определить») понятие множества. Не любое собрание элементов определённое каким-то свойством с потолка будет множеством. Если хочется прямо абсолютного определения, то такого вообще не существует ни для чего. Речь не идёт об аксиоматической теории множеств, но приведённого достаточно, чтобы более-менее представлять себе это понятие.
Аноним 22/02/15 Вск 00:11:08 #414 №240839 
>>240833
Арнольд, что ты тут делаешь, ты же умер.
Аноним 22/02/15 Вск 00:27:22 #415 №240844 
>>240838
>Существует натуральный ряд
>любое собрание элементов определённое каким-то свойством с потолка будет множеством
>множество ОПРЕДЕЛЯЕТСЯ целой кучей аксиом
>абсолютного определения... такого вообще не существует ни для чего

Уходи туда откуда пришел.
Аноним 22/02/15 Вск 00:29:34 #416 №240845 
>>240844
Скажи, что конкретно и почему тебе не нравится и я уйду.
Аноним 22/02/15 Вск 00:34:50 #417 №240847 
Подскажите хороший учебник на уровне школоты с мат. уклоном, где объяснялась бы комбинаторика.
Аноним 22/02/15 Вск 02:35:18 #418 №240900 
14245617185870.jpg
Сап матач, поясни по хардкору за школьника из России который математически доказал существование жизни после смерти.
Т.к. я гуманитарий, то в математике не шарю, но узнать что за йобу он там придумал интересно.
То что он придумал имеет какую-либо ценность?

http://www.youtube.com/watch?v=NrDe9O2odbw
Аноним 22/02/15 Вск 02:37:22 #419 №240901 
>>240847
"комбинаторика" виленкин
Аноним 22/02/15 Вск 02:43:25 #420 №240904 
>>240900
>То что он придумал имеет какую-либо ценность?
Нет.
inb4 пидора ответ
Аноним 22/02/15 Вск 09:55:20 #421 №240925 
14245881200940.gif
В чём различия при использовании в нейронной сети сигмоиды и гиперболического тангенса?
Аноним 22/02/15 Вск 10:52:18 #422 №240933 
>>240831

> Метакатегории, например.

Иди перечитывай определения, хуйло.

> Если ты мудак и не можешь понять самые основы, что с тобой говорить? Пацаны уже топосы гротендика ебашат, а ты все не можешь понять, что такое класс объектов.

Вот же долбаёпп...
Разговор про натуральные числа был, при чем тут класс???
Аноним 22/02/15 Вск 10:57:26 #423 №240934 
хз туда не туда
но я тут думала про функцию TREE
TREE(1) = 1
TREE(2) = 3
TREE(3) = ДОХУЯ

запилим нмерную модель вселенной
н = 0: либо в модели есть монада, либо нет. тогда со временем она так же и останется или так же ее не будет. каждый новый момент времени рождает 1 вселенную.
н = 1: у объекта на линии есть 3 варианта: не двигаться, двинуться "налево" или "направо". новый момент времени 3кол-во объектов новых вселенных
н = 2: у объекта есть дохуя вариантов: стоять на месте или двигаться в одном из дохуя направлений. новый момент времени - дохуя
кол-во объектов новых вселенных

я в том направлении думаю?
Аноним 22/02/15 Вск 11:47:36 #424 №240946 
>>240934
Что. Ты. Несёшь?
Аноним 22/02/15 Вск 12:15:19 #425 №240952 
>>240946

Откуда пошел этот уебанский стиль? На пиондосских форумах каждый второй три слова через точки пишет. Это типа новый SWAG YOLO?
Аноним 22/02/15 Вск 12:21:45 #426 №240953 
>>240934
>думала
Я хорошо разбираюсь в теме, запости сиськи с сапом и я тебе помогу.
Аноним 22/02/15 Вск 12:22:47 #427 №240954 
>>240934

Cфоткай свой анус с супом.

математик, к.ф-м.н., знающий решение
Аноним 22/02/15 Вск 12:33:37 #428 №240955 
>>240954
TY PERVIY
Аноним 22/02/15 Вск 14:01:41 #429 №240974 
14246029010260.png
Посаны, можете объяснить самый конец доказательства? Откуда мы знаем, что корни многочлена имеют вид lambda * I, почему корень не может быть каким-нибудь случайным оператором?
Аноним 22/02/15 Вск 14:21:33 #430 №240978 
>>240974
Ну, когда мы решаем обычные полиномиальные уравнения и находим их корни a1 a2 a3 .. an, то сам полином мы потом можем разложить как (x-a1)(x-a2)(x-a3)...(x-an), тут так же, решая это операторное уравнение находим набор лямбд, потом раскладываем наш операторные полином с их помощью.
Аноним 22/02/15 Вск 14:29:00 #431 №240981 
>>240925
бамп
Аноним 22/02/15 Вск 14:42:05 #432 №240983 
>>240745

бамп
Аноним 22/02/15 Вск 14:59:26 #433 №240989 
Бампать в тематике, пиздом
Аноним 22/02/15 Вск 17:16:46 #434 №241025 
>>240989
Иди нахуй. Бампается вопрос внутри треда, а не сам тред . Да и тред тоже можно бампать, если допускают правила доски.
Аноним 22/02/15 Вск 17:49:30 #435 №241030 
Помогите найти ошибку в рассуждениях:
Нужно найти количество способов выбрать два двухэлементных множества из шестиэлементного множества.
1) C26C24 - ну тут все очевидно
2) Убираем один элемент, потом еще один, потом выбираем первое множество из 4 оставшихся элементов, а остальные 2 попадут во второе множество:
6 5 C24
Почему рассуждения в (2) дают неправильный ответ?
Аноним 22/02/15 Вск 17:50:25 #436 №241031 
>>241030
Обосрался с разметкой и звездочка съелась, но вроде все понятно.
Аноним 22/02/15 Вск 18:18:53 #437 №241040 
>>240974
Потому что формальную переменную x в кольце C[x] мы можем заменить на переменную T если нам захочется. Ну или: из-за тривиального изоморфизма колец, если так угодно.

>>241030
Потому что ты забыл разделить на два, ведь свои два элемента ты можешь убирать в разном
порядке.
Аноним 22/02/15 Вск 18:27:30 #438 №241045 
Аноны, помогите с решением. 3 кролика, 4 ящика. Случайно распределили кроликов по ящикам. Какова вероятность что в ящике Х не будет ни одного кролика? Почему ответ 1/2
Аноним 22/02/15 Вск 18:29:54 #439 №241046 
>>241045
Все, дошло, не 1/2
Аноним 22/02/15 Вск 18:35:50 #440 №241047 
>>241025
Чо такой злой?
sageАноним 22/02/15 Вск 18:46:43 #441 №241050 
>>241045
https://ru.wikipedia.org/wiki/%D0%9F%D1%80%D0%B8%D0%BD%D1%86%D0%B8%D0%BF_%D0%94%D0%B8%D1%80%D0%B8%D1%85%D0%BB%D0%B5_(%D0%BA%D0%BE%D0%BC%D0%B1%D0%B8%D0%BD%D0%B0%D1%82%D0%BE%D1%80%D0%B8%D0%BA%D0%B0)
Аноним 22/02/15 Вск 18:49:08 #442 №241051 
>>241050
Очень содержательно
Аноним 22/02/15 Вск 19:31:42 #443 №241076 
>>240933
>Иди перечитывай определения, хуйло.
Ничего не буду перечитывать. Метакатегории не содержат множеств в опредлении.
Аноним 22/02/15 Вск 20:13:09 #444 №241085 
>>240750
В том, что там нужно уметь считать круговые многочлены и понимать, как они устроены (сходу могу назвать теоремы четыре крутые, в которых это ключевая часть). В таком контексте задача важная и ОЧЕВИДНАЯ. А метод кронекера это бессмысленная вычислительная хуита, которая ничего кроме ответа в частном случае не даст, а в данном случае еще и будет гораздо более сложным способом. Давай я тебе переформулирую задачу, чтобы ты успокоился со своими переборными решениями? Найти делитель 1+x^p+x^(2p)+...+x^(p-1)q, где p и q какие-то числа простые.
Аноним 22/02/15 Вск 20:44:09 #445 №241098 
>>241045
(3/4)3
Аноним 22/02/15 Вск 20:45:43 #446 №241100 
>>241076

Хуйло пиздливое, я тебе еще раз говорю, читай определение. Там с первых строк идёт "пусть имеются объекты a, b, ...".

Аноним 22/02/15 Вск 20:55:10 #447 №241105 
>>241098
Не)
Аноним 22/02/15 Вск 20:57:36 #448 №241106 
>>241105
В смысле не?
Аноним 22/02/15 Вск 20:57:36 #449 №241107 
>>241105
А сколько тогда?
Аноним 22/02/15 Вск 21:26:18 #450 №241123 
>>241100
И что? Где там используются множества?
collection of objects это не множество.
Аноним 22/02/15 Вск 21:38:18 #451 №241125 
>>241123

Сказочный долбоёб.
Аноним 22/02/15 Вск 21:59:41 #452 №241129 
>>241125
Шизик, ты пока что-то только оскорбляешь всех и обсираешься, таких обычно обходят стороной, но все-таки люди уделяют тебе внимание. По факту есть что сказать?
Аноним 22/02/15 Вск 22:19:12 #453 №241134 
>>241129

Не всех, а только тебя, тупая МРАЗЬ. Потому что ты пишешь несвязную хуйню, нахватался модных терминов - а что они означют не понимаешь.
Аноним 22/02/15 Вск 22:19:56 #454 №241135 
>>241129

И, да, подонок. Посмотрни кто тут оскорблять начал, а потом пизди.
Аноним 22/02/15 Вск 22:36:27 #455 №241136 
>>241134
Ты сначала разберись в том, что такое множество и чем оно отличается от класса, а потом других упрекай.
Аноним 23/02/15 Пнд 01:16:16 #456 №241199 
>>241134
>>241135
Ш И З И К
И
З
И
К
Аноним 23/02/15 Пнд 01:31:16 #457 №241208 
>>241045
Я, возможно, сейчас обосрусь, но, по моему, так.
Вероятность = (число благоприятных способов размещения)/(число всех способов размещения).

Чисто всех способов разместить 3 объекта по 4 ящикам = (4 варианта разместить 1-й в один из четырех ящиков) x (3 варианта разместить 2-й в один из оставшимся трех ящиков) x (2 варианта разместить 3-й в один из оставшихся двух ящиков) = 24 способа.

Число благоприятных способов размещения - это число способов разместить трех кроликов по трем ящикам, кроме ящика X. Это 3!=6 способов.

Итого получаем ответ: 6/24=1/4.
Аноним 23/02/15 Пнд 01:38:03 #458 №241216 
>>241208
ты гумонитарий?
Аноним 23/02/15 Пнд 01:40:14 #459 №241218 
>>241208
У меня за миллион попыток вероятность составила 0.316971.
Аноним 23/02/15 Пнд 01:41:42 #460 №241220 
>>241216
Не угадал.
Ты же не хочешь получить в рожу пространство элементарных событий, сигма-алгебру борелевских множеств и колмогоровскую вероятностную меру? Тем более что я теорвер проходил хуй знает сколько лет назад и мог сфейлить.
так что по существу давай.
Аноним 23/02/15 Пнд 01:51:42 #461 №241226 
>>241220
Я точного ответа не знаю, но на мой взгляд верный ответ (3/4)^3. 75% что попадет не в нужный ящик и так 3 раза
sageАноним 23/02/15 Пнд 02:22:04 #462 №241237 
>>241208
это верно
Аноним 23/02/15 Пнд 02:24:44 #463 №241240 
14246474848780.jpg
>>241218
>У меня за миллион попыток вероятность составила 0.316971.
Ты где-то сфэйлил, мой результат 1/4 подтверждает быдлокод в Mathematica уже за 100 тыс. попыток.

>>241226
>75% что попадет не в нужный ящик и так 3 раза
Нет, с чего бы.

>>241237
>это верно
Ок.
Аноним 23/02/15 Пнд 02:31:47 #464 №241242 
14246479072050.jpg
>>241045
Дебил,
ты не уточнил, может ли в одну ячейку попадать более одного кролика. В зависимости от этого ответ либо (3/4)^3, либо 1/4
Аноним 23/02/15 Пнд 02:38:09 #465 №241243 
>>241240
>Ты где-то сфэйлил
Скорее всего проблема не во мне, а в алгоритме псевдослучайности.
Аноним 23/02/15 Пнд 02:39:32 #466 №241244 
>>241242
тоже верно.
В первом случае условие непопадания будет как у тебя: (3/4)x(3/4)x(3/4)=0.75^3
Во втором случае (3/4)x(2/3)x(1/2)=1/4 Так еще проще, чем я сначала расписал

Но фраза "распределили кроликов по ящикам" в такого рода задачах обычно трактуется как "один объект - один ящик", так что, небось, тому анону в таком виде задачу и дали.
Аноним 23/02/15 Пнд 02:39:44 #467 №241245 
>>241242
Ну если нельзя, то это пиздец тупая задачка.
Аноним 23/02/15 Пнд 02:41:46 #468 №241247 
>>241243
А ты в чем писал?
Аноним 23/02/15 Пнд 02:42:09 #469 №241248 
>>241240
А лол, это я зафейлил. 0.316971 - вероятность для 4 кроликов.
Аноним 23/02/15 Пнд 02:46:25 #470 №241250 
14246487855350.png
14246487855351.png
>>241240
Вот для трех.
Аноним 23/02/15 Пнд 02:51:03 #471 №241252 
>>241250
Ну прально, (3/4)^3 для одной трактовки, 1/4 - для другой, выяснили ж уже.
Аноним 23/02/15 Пнд 03:01:09 #472 №241253 
Совместными усилиями двач решил мою задачу. Спасибо.
Аноним 23/02/15 Пнд 03:11:04 #473 №241256 
>>241253
Потом принесешь фотку пятерки в дневнике.
Аноним 23/02/15 Пнд 03:20:07 #474 №241259 
>>241256
Этот Кукембич порвался. Я ее вчера придумал, уточнить хотел ответ просто.
Аноним 23/02/15 Пнд 04:01:55 #475 №241262 
Пацаны, а есть программа, которая пробует брать пределы "по-человечески"? Т.е. алгоритмом перебора всяких алгебраических преобразований, нахождения знакомых бесконечно малых, замечательных пределов?

inb4: домашка
Аноним 23/02/15 Пнд 11:02:26 #476 №241303 
>>241262
В вольфраме альфа можно нажать step-by-step solution, но там вроде это только 2-3 раза в день можно делать, а потом нужен платный аккаунт.
Аноним 23/02/15 Пнд 11:13:02 #477 №241306 
>>240745

Бампую вопросы.

Тут есть настоящие математики или только решатели школьных домашек???
Аноним 23/02/15 Пнд 11:52:43 #478 №241317 
>>241306
Примем, что имеется достаточно сильная логика первого порядка, чтобы развить некоторую теорию моделей.

Мыслим две системы вещей.
1. Объекты.
2. Стрелки.

Мыслим следующие операции.
1. dom сопоставляет каждой стрелке объект.
2. cod сопоставляет каждой стрелке объект.
3. id сопоставляет каждому объекту стрелку.
4. композицию стрелок, которая сопоставляет двум стрелкам f и g таким, что dom g = cod f, стрелку g∘f.

Аксиомы таковы.
1. dom(id(a)) = a
2. cod(id(a)) = a
3. dom g∘f = dom f
4. cod g∘f = cod g
5. если можно взять композицию g∘f и композицию h∘(g∘f), то можно взять композиции h∘g и (h∘g)∘f, и причём h∘(g∘f) = (h∘g)∘f
6. если dom f = x, то id(x)∘f = f
7. если cod f = x, то f∘id(x) = f

Категорией называются любая модель для вышеназванного.

>Как можно определить понятие натурального числа через более элементарные понятия?
Никак, это уже аппаратное. Внутритеменная борозда теменной доли.
Аноним 23/02/15 Пнд 11:54:53 #479 №241318 
>>241317
то есть наоборот, ошибся с набором.
6. если cod f = x, то id(x)∘f = f
7. если dom f = x, то f∘id(x) = f
Аноним 23/02/15 Пнд 12:29:29 #480 №241328 
>>241317
Сейчас он скажет, что
> системы вещей
ЭТА МНОЖЫСТВА КОРОЧ ПРОСТ ДРУГИМЕ СЛОВАМЕ
Аноним 23/02/15 Пнд 12:42:19 #481 №241331 
>>241259
Только вот мамкины матиматике итт тебе неправильно нарешали, правильный ответ 1/2, считается просто по количествам исходов.
Аноним 23/02/15 Пнд 12:50:35 #482 №241334 
Вот задачка.
Есть 120 пятизначных чисел составленных из 1;2;3;4;5
то есть : 12345 12341.... ...54321.
Вопрос: как найти сумму?
Аноним 23/02/15 Пнд 13:05:25 #483 №241338 
>>241334
запрогать за 2 минуты, лол
ну или подумать 3 минуты о том, как складываются числа
Аноним 23/02/15 Пнд 13:11:03 #484 №241339 
>>241334
Каждая цифра на месте конкретного разряда стоит в 24 числах, значит сумма цифр в данном разряде по всем числам это 24(1+2+3+4+5).
Умножаешь это собственно на вклад, который даёт каждый разряд в сумму, и суммируешь по всем разрядам, получается 24
15*11111.
Аноним 23/02/15 Пнд 13:12:06 #485 №241340 
>>241339
Ну, ясно
> 24 x (1+2+3+4+5)
> 24 x 15 x 11111
Аноним 23/02/15 Пнд 13:40:11 #486 №241343 
>>240745
> определить натуральные числа
Индуктивные множества же!
Аноним 23/02/15 Пнд 14:49:17 #487 №241374 
>>241317

Определение системы вещей в студию.
Аноним 23/02/15 Пнд 14:51:27 #488 №241377 
>>241317

И еще вопрос, как развить эту логику первого порядка без множеств? Там как минимум набор символов нужен.
Аноним 23/02/15 Пнд 14:54:59 #489 №241379 
>>241317

> Мыслим следующие операции.

Т.е. если ты не привлекаешь теорию множеств, ты еще должен взять в качестве неопределямого понятия функцию? Круто.

> Никак, это уже аппаратное. Внутритеменная борозда теменной доли.

Тогда какой смысл строить формальную систему арифметики с точки зрени её обоснования? Она же ничего обосновывать не будет из-за рекурсии.
Аноним 23/02/15 Пнд 15:25:40 #490 №241383 
14246943405530.jpg
Поясните за быстрое умножение матриц с погрешностью? Какие прикладные методы используются для этого?

Аноним 23/02/15 Пнд 16:01:44 #491 №241390 
>>241331
Лол, ну давай, распиши нам количество исходов, заодно теорвер опровергнешь.
Аноним 23/02/15 Пнд 17:07:35 #492 №241406 
>>241328
Лол, ты угадал.
>Определение системы вещей в студию.
>И еще вопрос, как развить эту логику первого порядка без множеств
>Т.е. если ты не привлекаешь теорию множеств, ты еще должен взять в качестве неопределямого понятия функцию? Круто.

>>241343
Всем новозашедшим в тред. Вы общаетесь с ШИ ЗИ КОМ.

Основные шизотезисы:
1) Аксиома это скрытая рекурсия
2) Все на свете это множества, категории это множества
3) Математики тупые и занимаются хуйней, не замечая скрытой рекурсии и господство множеств
Аноним 23/02/15 Пнд 17:10:52 #493 №241408 
>>241331
Только за миллион попыток число приблизилось к 3/4 в кубе.
Аноним 23/02/15 Пнд 17:14:33 #494 №241410 
>>241047
Депрессия, а пить таблетки не хочу. На дваче отыгрываюсь.
◀tulp-hatcher▶ 23/02/15 Пнд 17:16:13 #495 №241411 
>>238758
>нужно просто жить иной жизнью
И что же ты под ней подразумеваешь?
Аноним 23/02/15 Пнд 17:34:55 #496 №241418 
>>241410
ну бля, попробуй курс паксила пропить, базарю еще захочешь
Аноним 23/02/15 Пнд 18:08:25 #497 №241424 
>>241374
Нам заданы два предиката, P(x) и Q(x).
Если P(x), то мы говорим, что x - объект.
Если Q(x), то мы говорим, что x - стрелка.

>>241377
Множество не есть совокупность вещей. Множество не есть набор символов. Множество всегда допускает какие-то операции над собой - например, выделение произвольных подмножеств, взятие пересечения и объединения с другими множествами. Набор символов таких операций не допускает. При построении логики мы, вообще говоря, не мыслим символы как некую единую известную нам совокупность. Мы просто принимаем, что в любой момент можем ввести символ, который не совпадает ни с одним ранее использованным нами символом. Ранее использованных нами символов всегда конечное количество, поэтому никаких проблем это предположение не вызывает. Обычно в качестве "нового символа" берётся буква x с достаточным количеством штрихов. Есть специальные науки, которые изучают это. Семиотика, например. Классические работы создал Фреге.

>>241379
Функция не есть операция; под операцией следует понимать функциональный символ. Функциональный символ - понятие из логики предикатов. Понятие функционального символа используется в том числе в ZFC, в аксиоме под названием "схема преобразования".
Я обосновал, что функциональный символ определен, когда предположил, что нам известна логика первого порядка. Функциональный символ довольно изящно определяют Бурбаки, если кому интересно.
Слово "функция" же чаще всего обозначает всем известную теоретико-множественную конструкцию.

>формальную систему арифметики
Под этим подразумевается некая формализация аппаратного умения считать. Аксиоматика Пеано есть уточнение тех действий, которые мы считаем дозволенными. Саму идею подсчёта аксиоматика Пеано не обосновывает.

>>241406
Нет, почему с шизиком. Нормальные вопросы для тех, кто в теме. Скрытая рекурсия - это вполне приличный термин, которым пользовался емнип Пуанкаре в своём трактате "О науке", причём пользовался именно в таком контексте, как твой собеседник, выстёбывая попытки аксиоматизировать натуральный ряд. Даже если этот анон - шизик, пока что ничего действительно безумного он не сказал.
Аноним 23/02/15 Пнд 18:39:52 #498 №241428 
>>241383
бамп, ну вы чего, теоретики?
Аноним 23/02/15 Пнд 19:00:05 #499 №241432 
>>241390
Количество рассовываний кроликов по 4м ящикам = количество рассовываний по одному в ящик + по 2 и 1 + по 3 кролика в ящик = 4 + 12 + 4 = 20.
Исключаем ящик - считаем то же самое для рассовываний по трём ящикам = 1 + 6 + 3 = 10.
10/20 = 1/2

>>241408
3/4^3 от 1/2 отличается на 5/64.
Аноним 23/02/15 Пнд 19:05:59 #500 №241435 
>>241432
>>241432
Ты не учел какие именно кролики.
Аноним 23/02/15 Пнд 19:10:40 #501 №241437 
>>241424
>Скрытая рекурсия - это вполне приличный термин, которым пользовался емнип Пуанкаре в своём трактате "О науке"
Ну так да, ведь за 100 лет в философии математики не было никого прогресса, этот вопрос больше не ставился, громких имен не было.
Аноним 23/02/15 Пнд 19:13:04 #502 №241438 
>>241432
>3/4^3 от 1/2 отличается на 5/64.
За 2 миллиона попыток погрешность только уменьшилась.
Аноним 23/02/15 Пнд 19:16:55 #503 №241439 
>>241432
http://theoryandpractice.ru/posts/7518-paradoks-probability
Аноним 23/02/15 Пнд 19:20:36 #504 №241440 
14247084365320.jpg
Саентач, такая хуйня. Пытюась осознать как выделять вектор признаков из изображения и нехуя не понимаю. Ведь изображение - это матрица NxM, в которой значения блядоинтенсивности пикселей (оттенки серого, мать их йоб), а в векторе признаков должны быть только 0 или 1 и он не должен быть дохуя большой.
Как это гавно работает? Я уже столько статей перечитал, а везде ебала типа мы ручками подобрали и у нас все заебись.
Machine Learning эскперты реквестируются в тред!
Плачу няшу авансом.
Аноним 23/02/15 Пнд 19:32:57 #505 №241444 
>>241435
Они не нумерованы, их и не надо учитывать.
Аноним 23/02/15 Пнд 19:36:37 #506 №241446 
>>240839

Алби Бэк.
Аноним 23/02/15 Пнд 20:25:49 #507 №241463 
>>240839
Он умер и попал в ад.
Аноним 23/02/15 Пнд 22:30:13 #508 №241506 
>>241432
>количество рассовываний по одному в ящик + по 2 и 1 + по 3 кролика в ящик

1) Зачем ты учитываешь рассовывание по одному и по два, когда в задаче рассовывают по три и ты уже это учел в последнем слагаемом?

2) Потом, учел ты это странно: ты считаешь всех кроликов тождественными, что ли, как элементаные частицы в квантовой статистике7 Лол. Классические объекты в стандартном теорвере по дефолту различимы, если не оговорено обратное. Но это даже неважно, потому что см. п.3
3) Допустим-таки, что неразличимы. Но тогда у тебя для полного числа вариантов рассовывания 3-х неразличимых кроликов по 4-м ящикам будет 4 варианта, как ты сам выше и написал (т.к. 4 возможности для оставшегося свободного ящика), а благоприятных вариантов - всего один (когда ровно нужный ящик свободен), так что все равно получаешь 1/4. То же будет, разумеется, если кролики различимы (просто числитель и знаменатель домножатся на 3! перестановок трех кроликов внутри занятых ими мест, а отношение 1/4 не поменяется) Получится как в >>241208

И да, ессно, здесь речь о варианте "один кролик - один ящик".
Аноним 23/02/15 Пнд 22:51:39 #509 №241510 
>>241506
>>241432
Стоп, щас тебя перечитал - ты походу решаешь для любого кол-ва кроликов в одном ящике. Ну блять, там же уже это обсудили - и даже давший задачу анон подтвердил, что правилен вариант 1/4, отвечающий варианту "один кролик - один ящик".
Аноним 23/02/15 Пнд 23:45:34 #510 №241535 
>>241510
В общем, резюмирую:
Если один "кролик - один ящик", то 1/4.
Если "до трех кроликов в одном ящике", то при неразличимых ("квантовых", лол) кроликах будет 1/2, при различимых (3/4)^3.
Аноним 24/02/15 Втр 03:05:31 #511 №241575 
>>237334
Забавненько школьнички поагрились на пасту.

По традиции, поясню по хардкору: паста раскрывает просто всю суть способа преподавания mathematics в росейских институтах образования, отсюда и лезут просто охуевшие от псевдосложности данной науки. Милые детишки, просто читайте правильные книжки, вот и всё. На английском, естественно. Рузке - в мусорку.


>>240304
>Сам учусь в 10 классе
ahaha, oh wow. /Sci was lost...
Аноним 24/02/15 Втр 09:45:47 #512 №241601 
14247603471460.jpg
>>241575
> Первокур нму на взлёте
> /sci was lost
Аноним 24/02/15 Втр 10:40:39 #513 №241607 
А это правда что в Америке ужасное мат. Образование?
Аноним 24/02/15 Втр 12:16:21 #514 №241620 
>>241607
Ага, они туууууупые))))0)0)))
Аноним 24/02/15 Втр 12:54:34 #515 №241624 
>>241575
>На английском, естественно. Рузке - в мусорку.
Лол, и переводы тоже в мусору, и вообще русский язык в мусорку.
Лоллирую с таких имбецилов.

мимо кфмн
Аноним 24/02/15 Втр 13:54:36 #516 №241631 
>>241624
>и вообще русский язык в мусорку
Именно так. Русский язык - мертворождённое говно. В русском уже нет работающих механизмов словообразования, поэтому люди, которые мыслят на русском языке, в массе не могут создавать ничего нового.
Единственное слово русского происхождения - осьминог. Все остальные слова русского языка заимствованы из других языков.
Аноним 24/02/15 Втр 14:20:08 #517 №241633 
>>241620
А нмушнек-вшешнек всё продолжал рваться.
Аноним 24/02/15 Втр 17:05:10 #518 №241656 
>>241633
Да я вообще мимопроходил. Это сарказм же.
Аноним 24/02/15 Втр 17:11:32 #519 №241658 
>>241607
В сравнении с рашкинским, шо то хуйня, шо то хуйня. В рашке школьная математика это вычисления, в сша школьная математика это более простые вычисления. Может сложиться впечатление, что в рашке все заебись, пока дело не переходит в вуз, где рашка сосет по всем показателям.
Аноним 24/02/15 Втр 17:35:58 #520 №241661 
>>241633
нму позорники
Аноним 24/02/15 Втр 17:49:20 #521 №241664 
>>241658
>где рашка сосет по всем показателям у двух - трёх престижнейших недешёвых заведений и даёт на клык всем остальным
Пофиксил.
Аноним 24/02/15 Втр 18:59:59 #522 №241678 
>>241664
>где две престижных заведения рашки дают на клык всем остальным, а остальные сосут даже у последних шараг сша
Пофиксил.
Аноним 24/02/15 Втр 19:03:09 #523 №241680 
Перекат >>241679
Аноним 24/02/15 Втр 20:45:06 #524 №241700 
>>241631
Сходи-ка на филфак, там на тебя тянки поссут.
Аноним 24/02/15 Втр 20:46:09 #525 №241701 
>>241678
>где две престижных заведения рашки дают на клык всем остальным, а остальные сосут у двух - трёх престижнейших недешёвых заведений и дают на клык всем остальным
Аноним 25/02/15 Срд 08:25:36 #526 №241779 
>>241701
>где все дают друг другу на клык забывая про математику
>и вновь и вновь дают друг другу на клык
Аноним 25/02/15 Срд 11:48:32 #527 №241791 
>>241575
> На английском, естественно. Рузке - в мусорку
Вот на русском годнота для школьников:
http://www.mccme.ru/free-books/yaschenko/v08book-08.pdf

Вообще, ты хуйню несешь, мне даже объяснять лень очевидные вещи.
Аноним 25/02/15 Срд 15:05:03 #528 №241806 
Как нынче в 57-й школе, кто-нибудь знает? Она по-прежнему эталон мат. образования школьников в России?
Аноним 25/02/15 Срд 21:44:40 #529 №241892 
>>241424

> Множество не есть совокупность вещей. Множество не есть набор символов...

Но по сути этим мы постулируем, что умеем свободно обращаться со счетными процессами, а значит и можем легко определить содержательные счетные множества (в том числе и со всем операцифми, почему нет?) и понятие функции в содержательном (а не как теоретико-множественное кодирование через график) смысле не выходя за рамки уже имеющихся логических средств.

Я говорю об общем здравом смысле, конечно можно придраться с позиции каких нибудь конструктивистов, которые законченную бесконечность вообще не признают.

Семиотика же математическая наука, её доводы могут представлять интерес для общего развития, но как серьёзное основание вряд ли.

> Под этим подразумевается некая формализация аппаратного умения считать. Аксиоматика Пеано есть уточнение тех действий, которые мы считаем дозволенными. Саму идею подсчёта аксиоматика Пеано не обосновывает.

Понятно. Жаль, что явно это мало где пишут, хотя с этого следовало бы каждый учебник по мат. логике начинать.

А ты где это прочитал, случайно не у Клини? Вообще, Может какие книжки хорошие посоветуешь по основаниям? А то я на русском ничего не нашел (кроме того же Клини), а на английском даже не знаю где искать.


Аноним 26/02/15 Чтв 00:47:49 #530 №241917 
>>241892
А в чём проблема вообще со множествами у кого-то? Зачем гнаться за каким-то мифическим способом определить всё без множеств, не в какой-то конкретной аксиоматике, а как ты их понимаешь - т.е. без взятия совокупности каких-либо сущностей вообще, который я даже не уверен что возможен вследствие самого характера человеческих рассуждений?
Аноним 26/02/15 Чтв 09:19:35 #531 №241944 
>>241917

Нет проблем, просто хочется всё упорядочить и выяснить какие понятия являются основными (необходимо неопределямымыи), а какие производными от них.
Аноним 26/02/15 Чтв 09:20:39 #532 №241945 
Опечатка:

> Семиотика же математическая наука

НЕ математическая.

comments powered by Disqus

Отзывы и предложения